Download as pdf or txt
Download as pdf or txt
You are on page 1of 78

IBPS 

PO (MAINS)­18/2022
Passage for Question 1, 2, 3, 4 (ST­2035 SBIPO MAINS R (16­20) 2018)
eng Passage

Study the following information carefully and answer the questions given below­

Ten people P, Q, R, S, T, U, V, W, X and Y are marketing executives of a mobile company in ten different cities i.e. A, B, C, D, E, F, G, H, I and J. They are sitting
around a rectangular table which has four seats each on the longer side and one seat each on the shorter side. All the persons are sitting at equal distances and
facing one of the persons sitting on the opposite side. They are sitting in such a way that four persons are facing north, four are facing south, one is facing west and
one is facing east.
Q and Y are not sitting on the same side of the table. Only one person is sitting between the executives from H and B on the same side of the table. R is not an
executive from G. Executive from J is the immediate neighbour of V and X and they are facing the south direction. V is sitting opposite R. X is sitting third to the left
of P on the same side of the table. S is sitting on the shorter side of the table and is sitting opposite the executive from D. The executive from G is facing north and is
the immediate neighbour of the executives from F and A on the same side of the table. Y is sitting opposite the executive from I and he is the immediate neighbour of
the executive from F. Executives from B and E are sitting diagonally opposite of each other. T who is facing east direction is sitting opposite executive from C. Q sits
third to the right of W. H is not sitting opposite W.

hi Passage

िन िलिखत जानकारी क ो यानपू व क प ढ़ए औ र द ए ग ए ो क ा उ र दीिजये­

दस ि P, Q, R, S, T, U, V, W, X और Y दस िविभ शहर अथात A, B, C, D, E, F, G, H, I और J म मोबाइल कंपनी के िवपणन अिधकारी ह। वे एक आयताकार मेज के
चार ओर बैठे ह, िजसक बड़ी भुजा म चार सीट ह और येक छोटी भुजा पर एक सीट है। सभी ि समान दूरी पर बैठे ह और िवपरीत भुजा पर बैठे ि य म से कसी
एक के सामने है। वे इस कार बैठे ह क चार ि उ र क ओर देख रहे ह, चार दि ण क तरफ, एक पि म क ओर और एक पूव क ओर देख रहा है।
Q और Y मेज क एक ही भुजा पर नह बैठे ह। केवल एक ि मेज के एक ही ओर H और B के अिधका रय के बीच बैठा है। R G का कायकारी नह है। J का कायकारी V
और X का त काल पड़ोसी है और वे दि ण दशा म मुख करके बैठे ह। V, R के सामने बैठा है। X मेज क एक ही भुजा पर P के बाय तीसरे थान पर है। S मेज क छोटी भुजा पर
बैठा ह और D से कायकारी के सामने बैठा है। G का कायकारी उ र क ओर मुख करके F और A के अिधका रय के तुर त पड़ोस म बैठा है और वे सब मेज के एक ही ओर ह। Y, I
से कायकारी के िवपरीत बैठा है और वह F के कायकारी अिधकारी का त काल पड़ोसी है। B और E के अिधकारी एक­दूसरे के सामने िवकणतः िवपरीत बैठे है। T, जो पूव दशा म
मुख करके बैठा है, C के कायकारी अिधकारी के सामने बैठा है। Q, W के दाय ओर तीसरे थान पर है। H, W के िवपरीत नह बैठा है।

Q­1 If we count clockwise, who is seated in the seventh place from T?
eng

01. V 02. W
03. U 04. S
05. Y
Q­1 य द हम दि णावत दशा म िगन, तो T से सातव थान पर कौन बैठा है?
hi

01. V 02. W
03. U 04. S
05. Y
Solution for Question 1 Ans Key: 3
eng

hi

Q­2 Which of the following pairs is different from the others?
eng
01. P ­ E 02. U ­ J
03. X ­ A 04. S ­ F 
05. R ­ H
Q­2 िन िलिखत म से कौन सा यु म अ य से िभ है?
hi

01. P ­ E 02. U ­ J


03. X ­ A 04. S ­ F 
05. R ­ H
Solution for Question 2 Ans Key: 4
eng

hi

Q­3 Who is the marketing executive from E?
eng

01. P 02. T
03. U 04. X
05. Y
Q­3 E से िवपणन कायकारी कौन है?
hi

01. P 02. T
03. U 04. X
05. Y
Solution for Question 3 Ans Key: 5
eng

hi

Q­4 Who is seated second to the right of executive from J?
eng

01. P 02. R
03. T 04. S
05. W
Q­4 J के कायकारी अिधकारी के दाएं दूसरा कौन बैठा है?
hi
01. P 02. R
03. T 04. S
05. W
Solution for Question 4 Ans Key: 1
eng

hi

Passage for Question 5, 6, 7, 8 (ST­2037 SBIPO MAINS R (1­6) 2018)
eng Passage

A string of numbers is given as input. The further steps given are obtained by applying certain logic. Numbers of step II have been obtained by using at least one
digit of each number in step I. Each step is a result of the previous step only.

hi Passage

सं या क एक शृंखला इनपुट के प म दी जाती है। आगे के दए गए कुछ चरण कुछ तक लागू करके ा कए जाते ह। चरण I म येक सं या के कम से कम एक अंक का
उपयोग करके चरण II क सं या ा क गई है। येक चरण केवल िपछले चरण का प रणाम है।

Q­5 If in the first step, the second numbers of each box are added together and then multiplied by 8 then what will be the resultant value?
eng

01. 472 02. 456


03. 654 04. 532
05. 534
Q­5 य द पहले चरण म,  येक बॉ स क दूसरी सं या को आपस म जोड़ दया जाये और फर उसे 8 से गुणा कया जाये तो प रणामी मान या होगा?
hi

01. 472 02. 456


03. 654 04. 532
05. 534
Solution for Question 5 Ans Key: 1
eng

34+8+17 = 59 x 8 = 472

hi

34+8+17 = 59 x 8 = 472

Q­6 What is the difference between the second number of the first box and the first number of the second box from the left end in step II?
eng
01. 237 02. 210
03. 234 04. 215
05. 207
Q­6 चरण II म बाएं छोर से पहले बॉ स क दूसरी सं या और दूसरे बॉ स क पहली सं या के बीच अंतर या है?
hi

01. 237 02. 210


03. 234 04. 215
05. 207
Solution for Question 6 Ans Key: 2
eng

272 – 62 = 210
hi

272 – 62 = 210

Q­7 What will be the result when all the digits are multiplied in step III?
eng
01. 0 02. 1
03. 3 04. 8
05. 7
Q­7 प रणाम या होगा जब चरण III म सभी अंक का आपस म गुणा कर दया जाता है?
hi

01. 0 02. 1
03. 3 04. 8
05. 7
Solution for Question 7 Ans Key: 1
eng

4x7x5x2x1x0 = 0

hi

4x7x5x2x1x0 = 0

Q­8 Which of the following combinations represent the first number of the second box and the second number of the first box from the left end in step I?
eng

01. 41, 58 02. 41, 34


03. 25, 34 04. 41, 10
05. 25, 8
Q­8 चरण I म बाय छोर से दूसरे बॉ स क पहली सं या और पहले बॉ स क दूसरी सं या का ितिनिध व िन म से कौन­सा संयोजन करता है?
hi

01. 41, 58 02. 41, 34


03. 25, 34 04. 41, 10
05. 25, 8
Solution for Question 8 Ans Key: 3
eng

hi

Passage for Question 9, 10, 11, 12 (ST­2038 SBIBCL MAINS R (21­25) 2018)
eng Passage

Study the following information carefully and answer the questions given below.
In a certain code language “just pray for good health” is written as “24, 29, 21, 08, 14”, have bad nature” is written as “09, 07, 11”, “for new life” is written as “21,
26, 09” and “pray human” is written as “ 29, 19”.

hi Passage

िन िलिखत व था क ा यानपू व क अ ययन कर औ र नीचे द ए ग ए के उ र द :

एक िनि त कोड भाषा म "just pray for good health” को "24, 29, 21, 08, 14”, have bad nature” को "09, 07, 11”, “for new life” को “21, 26, 09” और "pray human” को


"29, 19” िलखा जाता है।

Q­9 “12, 32” is stand for­
eng

01. great idea 02. costly fare 


03. high level 04. good novel
05. with honesty
Q­9 “12, 32” का अथ है­
hi

01. great idea 02. costly fare 


03. high level 04. good novel
05. with honesty
Solution for Question 9 Ans Key: 5
eng Rules for knowing the code = Place value of the last letter of each word + total number of letters in each word
hi कोड ात करने के िलए िनयम =  येक श द के अंितम अ र का थानीमान+  येक श द म अ र क कुल सं या

Q­10 What is the code for “Remote” in that code language?
eng

01. 11 02. 15
03. 10 04. 25
05. 29
Q­10 उसी कोड भाषा म " Remote” को कैसे िलखा जायेगा?
hi

01. 11 02. 15
03. 10 04. 25
05. 29
Solution for Question 10 Ans Key: 1
eng Rules for knowing the code = Place value of the last letter of each word + total number of letters in each word.

hi कोड ात करने के िलए िनयम =  येक श द के अंितम अ र का थानीमान+  येक श द म अ र क कुल सं या

Q­11 If 25 is related to “court” and 16 is related to “zeal” then 25 will be related to which option?
eng

01. Flame 02. Focus


03. Fight 04. Follow
05. Fame
Q­11 य द 25 "court" से संबिं धत है और 16 "zeal" से संबिं धत है तो 25  कस िवक प से संबिं धत होगा?
hi

01. Flame 02. Focus


03. Fight 04. Follow
05. Fame
Solution for Question 11 Ans Key: 3
eng Rules for knowing the code = Place value of the last letter of each word + total number of letters in each word
hi कोड ात करने के िलए िनयम =  येक श द के अंितम अ र का थानीमान+  येक श द म अ र क कुल सं या
Q­12 How is “Solution” written in that code language?
eng

01. 22 02. 25
03. 30 04. 35
05. 19
Q­12 उसी कोड भाषा म "Solution” को कैसे िलखा जायेगा?
hi

01. 22 02. 25
03. 30 04. 35
05. 19
Solution for Question 12 Ans Key: 1
eng Rules for knowing the code = Place value of the last letter of each word + total number of letters in each word
hi कोड ात करने के िलए िनयम = येक श द के अंितम अ र का थानीमान+ येक श द म अ र क कुल सं या

Passage for Question 13, 14, 15, 16 (ST­2040 SBIBCL MAINS (6­10) 2018)
eng Passage

Study the following information carefully and answer the questions given below:

Seven friends are A, Y, C, W, E, U and T. They belong to seven different companies i.e. 16, 17, 18, 19, 20, 21 and 22. They live in different cities, Varanasi,
Mumbai, Bangalore, Hyderabad, Chennai, Indore and Bhopal, but not necessarily in the same order.
A belongs to company 22 and does not live in Indore and Mumbai. 
T lives in Bangalore. One who lives in Hyderabad belongs to company 20. 
The one who belongs to company 17 lives in Bangalore.
W neither belongs to company 21 nor 16 but he lives in Mumbai. 
E lives in Bhopal and belongs to company 19. 
Y neither live in Varanasi nor Indore. 
The one who lives in Indore belongs to company 16.
U belongs to either company 20 or 18 and live Hyderabad.

hi Passage

िन िलिखत जानकारी क ो यानपू व क अ ययन करे औ र नीचे दये गये के उ र दीिजए:

सात िम A, Y, C, W, E, U और T अलग­अलग कंपिनय जैस­े 16, 17, 18, 19, 20, 21 और 22 से स बंिधत है। उनमे से येक सात अलग­अलग शहर वाराणसी, मुब
ं ई,
बगलोर, हैदराबाद, चे ई, इंदौर और भोपाल म रहते ह, ले कन ज री नह क इसी म म हो।
A कंपनी 22 से स बंिधत है और इंदौर और मुब
ं ई म नह रहता है।
T बगलोर म रहता है। वह ि जो हैदराबाद म रहता है कंपनी 20 से स बंिधत है।
वह ि जो कंपनी 17 से स बंिधत है बगलोर म रहता है।
W न तो कंपनी 21 न ही 16 से स बंिधत है और वह मुब
ं ई म रहता है।
E भोपाल म रहता है और कंपनी 19 से स बंिधत है।
Y न तो वाराणसी न ही इंदौर म रहता है।
जो ि इंदौर म रहता है वह कंपनी 16 से स बंिधत है।
U या तो कंपनी 20 या 18 से स बंिधत है और हैदराबाद म रहता है।

Q­13 C belongs to which city and company?
eng

01. Mumbai, 19 02. Bangalore, 20


03. Hyderabad, 22 04. Chennai, 17
05. Indore, 16
Q­13 C  कस शहर और कंपनी से संबिं धत है?
hi

01. मुब
ं ई, 19 02. बगलोर, 20
03. हैदराबाद, 22 04. चे ई, 17
05. इंदौर, 16
Solution for Question 13 Ans Key: 5
eng
hi

Q­14 Who belongs to Mumbai?
eng

01. T 02. E
03. U 04. W
05. C
Q­14 मुब
ं ई से कौन संबिं धत है?
hi

01. T 02. E
03. U 04. W
05. C
Solution for Question 14 Ans Key: 4
eng

hi

Q­15 Four of the following five are alike in a certain way and so form a group. Which one does not belong to that group?
eng

01. A – Company 18 02. W ­ Company 19


03. C ­ Company 21 04. U ­ Company 20
05. T ­ Company 16
Q­15 िन िलिखत पांच म से चार एक िनि त तरीके से समान ह और इसिलए एक समूह बनाते ह। कौन सा समूह उस समूह से संबिं धत नह है?
hi

01. A – कंपनी 18 02. W ­ कंपनी 19


03. C ­ कंपनी 21 04. U ­ कंपनी 20
05. T ­ कंपनी 16
Solution for Question 15 Ans Key: 4
eng
hi

Q­16 If person A change his company and city with Y then he belongs to which city and company?
eng

01. Varanasi, 22 02. Bangalore, 20


03. Hyderabad, 17 04. Chennai, 21
05. Indore, 16
Q­16 य द ि A, Y के साथ अपनी कंपनी और शहर बदलता है तो वह कस शहर और कंपनी से संबिं धत है?
hi

01. वाराणसी, 22 02. बगलोर, 20


03. हैदराबाद, 17 04. चे ई, 21
05. इंदौर, 16
Solution for Question 16 Ans Key: 4
eng

hi

Passage for Question 17, 18, 19, 20, 21, 22 (ST­2025 SBI BCL MAINS (11­15)R 2019)
eng Passage

Study the given information carefully and answer the following questions.

Statement: Yoga, the Ayurvedic remedy for every illness whether tension or cancer.  Adults and children are getting slothfully busy with Tech devices; seeing the
present condition the day to day maneuver, yoga is incumbent. It is a meticulous pursuit that gives an active life from passive life; dynamic life from static life.
Alternatively, chemicals are balefully upshot the physical strength, for that time being it can provide relief to the body but not worthy at all. Medicines are also
important as yoga’s effect is very time taking and some health problems need instant recovery. For the long term, yoga is highly effective and can’t be replaced. Not
a day to be referred a yoga day but it should be practiced regularly.

(A) Yoga, the universal medicine for all the remedies, gets control over the body and mind which makes the individual’s activities more worthy and healthy.
(B) Untrained instructions can adversely affect the body and trained instructor is hard to find. 
(C) Mental relaxation is very important for concentration that can only be obtained by practicing yoga.
(D) Natural cure is better than artificial medication.
(E) Schools and colleges should make yoga mandatory in their curriculum which will make today's generation regular to it and safe from further illness.

hi Passage

िन िलिखत जानकारी का यानपूवक अ ययन करे और नीचे दए गए के उ र दीिजये:


कथन: योगा !  हर बीमारी के िलए आयुव दक उपचार ­चाहे तनाव हो या कसर। आजकल बड़े और ब े टेक उपकरण के साथ आलसी हो रहे है;  दनचया क यही दशा देख कर
योग ज री है। यह एक सावधानीपूवक दनचया है जो िनि य जीवन से स य जीवन देता है;  थैितक जीवन से गितशील जीवन देता है। वैकि पक प से, रासायिनक
शारी रक शि को कम कर रहे ह,  य क उस समय यह शरीर को राहत दान कर सकता है ले कन िबलकुल यो य नह है। दवाएं भी मह वपूण ह य क योग का भाव ब त
समय ले लेता है और कुछ वा य सम या को तुरत ं इलाज क आव यकता होती है। लंबी अविध के िलए, योग का भाव सबसे भावी है और इसे ित थािपत नह कया जा
सकता है। एक दन को योग दवस नह कहा जाना चािहए ले कन इसे िनयिमत प से अ यास कया जाना चािहए।

(A) योग, सभी उपचार के िलए सावभौिमक दवा, शरीर और दमाग पर िनयं ण ा करता है जो ि क गितिविधय को अिधक यो य बनाती है और वा य को भी व थ
करता है।
(B) अनचाहे िनदश िवपरीत प से भािवत कर सकते है और िशि त िश क आज के समाज म ब त ही मुि कल से िमलते ह।
(C) दुिनया या कसी भी काम पर यान क त करने म मानिसक िव ाम ब त मह वपूण है, िजसे केवल योग करके ही ा कया जा सकता है।
(D) कृि म दवा से ाकृितक इलाज बेहतर होता है।
(E)  कूल और कॉलेज को अपने पा म म योग को अिनवाय बनाना चािहए जो आज क पीढ़ी को िनयिमत प से आगे बढ़ाएगा और आगे क बीमारी से सुरि त रखेगा।
Q­17 Which of the following will be the assumption for the above statement?
eng

01. D 02. A and C


03. C and B 04. E
05. None of these
Q­17 उपरो कथन के िलए कौन सा पूवधारणा होगी?
hi

01. D 02. A और C


03. C और B 04. E
05. इनम से कोई नह
Ans Key: 1

Q­18 Which of the following will be inferred from the above statement?
eng

01. C and A 02. E Or B


03. D and A 04. B
05. None of these
Q­18 उपरो दए गए बयान से िन िलिखत म से कौन सा िन कष होगा?
hi

01. C और A 02. E या B


03. D और A 04. B
05. इनम से कोई नह
Ans Key: 3

Q­19 Which of the following will be the course of action for the above statement?
eng

01. A 02. C or A


03. D and A 04. Only E
05. None of these
Q­19 उपरो दए गए बयान के िलए िन िलिखत म से कौन सी कायवाही होगी?
hi

01. A 02. C या A


03. D और A 04. केवल E
05. इनम से कोई नह
Ans Key: 4

Q­20 Which of the following will be the strong argument of the above statement?
eng

01. D or A 02. E


03. B 04. C and B
05. None of these
Q­20 िन िलिखत म से कौन सा उपयु बयान का मजबूत तक होगा?
hi

01. D या A 02. E


03. B 04. C और B
05. इनम से कोई नह
Ans Key: 3

Q­21 Which of the following will be the weak argument of the above Statement?
eng

01. B or D 02. Only C


03. A and  B 04. Only E
05. None of these
Q­21 िन िलिखत म से कौन सा उपयु कथन का कमजोर तक होगा?
hi

01. B या D 02. केवल C


03. A और B 04. केवल E
05. इनम से कोई नह
Ans Key: 2

Q­22 Which of the following will be the effect of the above statement?
eng

01. Only A 02. C


03. D and E 04. B
05. None of these
Q­22 उपरो म से कौन­सा दए गए कथन का भाव होगा ?
hi

01. Only A 02. C


03. D और E 04. B
05. इनम से कोई नह
Ans Key: 1

Passage for Question 23, 24, 25, 26, 27 (ST­2069CRPPOMAINS(1­5R)2019)
eng Passage

Study the following information carefully and answer the questions given below:

Ten persons from different countries Canada, Cuba, Japan, France, Angola, Bhutan, China, Sri Lanka, Denmark and Russia– are sitting in two parallel rows
containing five people each in such a way that there is an equal distance between adjacent persons. In row 1 A, B, C, D and E are seated and all of them are facing
north. In row 2, M, N, O, P and Q are seated and all of them are facing south. Therefore, in the given seating arrangement, each member is seated in a row facing
another member of the other row. 
C sits third to the right of the person from Russia. O is an immediate left of the person who faces the person from Russia. Only one person sits between O and the
person from France. The person from France faces the one is an immediate neighbour of the person from Japan. The person from Canada faces the person from
Bhutan. O is not from Bhutan. M does not face the person from Russia. N faces the person from Angola. The one who faces P sits third to the left of A. B sit at one
of the extreme ends of the line. The person from Russia neither faces the person from Cuba nor immediate neighbour of the person from Cuba. The person from Sri
Lanka is facing the one is second to the left of E. D is not an immediate neighbour of the one who is from Denmark.

hi Passage

िन िलिखत जानकारी क ा यानपू व क अ ययन करे औ र नीचे द ए ग ए के उ र दीिजये:

िविभ देश कनाडा,  यूबा, जापान,  ांस, अंगोला, भूटान, चीन,  ीलंका, डेनमाक और स के दस ि दो समानांतर पंि य म पांच­पांच लोग के साथ इस तरह से बैठे ह
क आस­पास के ि य के बीच समान दूरी है। पंि  1 म A, B, C, D और E बैठे ह और वे सभी उ र क ओर देख रहे ह। पंि  2 म M, N, O, P और Q बैठे ह और वे सभी
दि ण क ओर देख रहे ह। इसिलए, इस बैठक व था म येक पंि म बैठे सद य का मुख दूसरी पंि म बैठे अ य सद य क ओर है।
C,  स से संबि धत ि के दाय ओर तीसरा बैठता है। O उस ि के तुरत ं बाय ओर है जो स से संबि धत ि क ओर देख रहा है। O और ांस से संबि धत ि के बीच
म केवल एक ि बैठता है। ांस से संबि धत ि उस ि क ओर देख रहा है जो जापान से संबि धत ि का तुरत ं पड़ोसी है। कनाडा से संबि धत ि , भूटान से
संबि धत ि क ओर देख रहा है। O भूटान से नह है। M,  स से संबि धत ि क ओर नह देख रहा है। N, अंगोला से संबि धत ि क ओर देख रहा है। िजस ि का
मुख P क ओर है, A के बाय ओर तीसरा बैठता है। B पंि के अंितम छोर पर है। स से संबि धत ि , न तो यूबा से संबि धत ि क ओर देख रहा है और न ही यूबा से
संबि धत ि का तुरत ं पड़ोसी है। ीलंका से संबि धत ि ,उस ि क ओर देख रहा है जो E के बाएं से दूसरे थान पर है। D, डेनमाक से संबि धत ि का तुरत ं पड़ोसी
नह है।

Q­23 Who amongst the following faces the person from Russia?
eng

01. The person from Denmark 02. M


03. O 04. The person from Cuba
05. The person from Japan
Q­23 िन िलिखत म से कौन स से संबि धत ि क ओर देख रहा है?
hi

01. डेनमाक का ि 02. M


03. O 04. यूबा का ि
05. जापान का ि
Solution for Question 23 Ans Key: 1
eng
hi

Q­24 Who is sitting between O and the person from France?
eng

01. The person from Bhutan 02. Q


03. N 04. The person from Cuba
05. P
Q­24 O और ांस से संबि धत ि के बीच म कौन बैठा है?
hi

01. भूटान का ि 02. Q


03. N 04. यूबा का ि
05. P
Solution for Question 24 Ans Key: 4
eng

hi

Q­25 Who amongst the following sit at extreme ends of the rows?
eng

01. D and the person from France 02. The persons from Canada and Denmark


03. The person from Angola and M 04. The person from Cuba and D
05. C and N
Q­25 िन िलिखत म से कौन पंि य के अंितम छोर पर बैठता है?
hi

01. D और ांस का ि 02. कनाडा और डेनमाक के ि


03. अंगोला का ि और M 04. यूबा का ि और D
05. C और N
Solution for Question 25 Ans Key: 5
eng

hi

Q­26 B is from which of the following country?
eng

01. Canada 02. China


03. Angola 04. Bhutan
05. Japan
Q­26 B िन िलिखत म से कस देश से है?
hi

01. कनाडा 02. चीन


03. अंगोला 04. भूटान
05. जापान
Solution for Question 26 Ans Key: 2

eng

hi

Q­27 Who amongst the following is from Cuba?
eng

01. C 02. O
03. M 04. D
05. A
Q­27 िन िलिखत म से कौन यूबा से है?
hi

01. C 02. O
03. M 04. D
05. A
Solution for Question 27 Ans Key: 3
eng

hi

Passage for Question 28, 29, 30, 31, 32 (ST­2069CRPPOMAINS(16­20R)2019)
eng Passage

Study the following information carefully and answer the questions given below:

L, M, N, O, P, Q, R, S, T, U, V, W, X, Y and Z live on five different floors from one to five. Ground floor is number 1 and top floor is number 5, but not
necessarily in the same order. There are three flats on each floor­ flat­1, flat­2 and flat­3 from west to east respectively such that flat­1 of third floor is exactly above
flat­1 of second floor which is exactly above flat­1 of first floor and other flats are placed in the same way. 
R lives in an even numbered flat. X lives just above U in a flat numbered 3. R does not live on the same floor on which X lives. W lives just above Y who is one
floor above to N in same numbered flat. Y does not live on the same floor on which X and L lives. V lives just below U in same numbered flat. T lives to the left of
Z who is to the left of O in the same floor. R lives two floors above to L in same numbered flat. Q lives just above to S but in different numbered flat. M lives just
right of P in the same floor but not in odd numbered floor.

hi Passage

िन िलिखत जानकारी क ा यानपू व क अ ययन करे औ र नीचे द ए ग ए के उ र दीिजये:

L, M, N, O, P, Q, R, S, T, U, V, W, X, Y और Z एक से पांच तक पांच अलग­अलग मंिजल पर रहते ह। ाउंड लोर नंबर 1 है और टॉप लोर नंबर 5 है, ले कन ज री नह
क इसी म म हो। येक मंिजल पर तीन लैट ह­  लैट ­1,  लैट ­2 और लैट ­3 जो मशः पि म से पूव क ओर ह जैसे तीसरी मंिजल का लैट ­1 दूसरी मंिजल के लैट ­1
से ठीक ऊपर है जो पहली मंिजल के लैट ­1 से ठीक ऊपर है और अ य लैट का उसी तरह है।
R एक सम सं या वाले लैट म रहता है। X,  लैट नंबर 3 म U के ठीक ऊपर रहता है। R उस मंिजल पर नह रहता है िजस पर X रहता है। समान सं या वाले लैट म W, Y के
ठीक ऊपर रहता है, जो N से एक मंिजल ऊपर है। Y उस मंिजल पर नह रहता िजस पर X और L रहते ह। समान सं या वाले लैट म V, U से ठीक नीचे रहता है। एक ही मंिजल
म T, Z के बाय रहता है, जो O के बाय रहता है। समान सं या वाले लैट म R, L से दो तल ऊपर रहता है। Q, S के ठीक ऊपर रहता है ले कन दोन अलग­अलग नंबर वाले लैट
म रहते ह। एक ही मंिजल म M, P के ठीक दाय रहता है ल कन िवषम सं या वाली मंिजल पर नही रहता है।
Q­28 How many persons live below U?
eng

01. Three 02. Six


03. Nine 04. Five
05. None of these
Q­28 U के नीचे कतने ि रहते ह?
hi

01. तीन 02. छह


03. नौ 04. पांच
05. इनम से कोई नह
Solution for Question 28 Ans Key: 2
eng

hi

Q­29 Four of the following five are alike in a certain way and so from a group. Which one does not belongs to that group?
eng

01. Z 02. M
03. Q 04. L
05. N
Q­29 दी गए व था के आधार पर िन िलिखत पांच म से चार कसी कार से समान ह इसिलए उनका एक समूह बनता है वह कौन सा है जो इस समूह म नह आता है?
hi

01. Z 02. M
03. Q 04. L
05. N
Solution for Question 29 Ans Key: 5
eng

hi

Q­30 Who among the following lives immediate left to X? 
eng

01. Z 02. T
03. W 04. M
05. None of these
Q­30 िन िलिखत म से कौन X के तुरत
ं बाय है?
hi

01. Z 02. T
03. W 04. M
05. इनम से कोई नह
Solution for Question 30 Ans Key: 4
eng

hi

Q­31 Which of the following group of person lives on same floor?
eng

01. T, Z, X 02. Y, L, V


03. Y, Q, V 04. N, V, U
05. W, M, O
Q­31 िन म से कौन सा समान मंिजल पर रहने वाल के सही समूह को दशाता है?
hi

01. T, Z, X 02. Y, L, V


03. Y, Q, V 04. N, V, U
05. W, M, O
Solution for Question 31 Ans Key: 3
eng

hi

Q­32 Who lives in flat­3 of fifth floor?
eng

01. R 02. S
03. O 04. T
05. None of these
Q­32 पाँचव मंिजल के लैट ­3 म कौन रहता है?
hi

01. R 02. S
03. O 04. T
05. इनम से कोई नह
Solution for Question 32 Ans Key: 3

eng

hi

Passage for Question 33, 34, 35, 36 (ST­2069CRPPOMAINS(21­25R)2019)
eng Passage

Study the following information carefully and answer the questions given below:

There are PQ axis in such a way that P is in north and Q is in south direction. There is AB axis in such a way that A is in west direction and B is in east direction.
PQ axis and AB axis intersect at a point Z in such a way that PZ is 25m, ZQ is 21m, ZA is 19m, ZB is 18 m. Atul starts from point Q and walks 20m in south
direction and then he turns right and walks 29 m. Rahul starts from point B and walks 15 m in north direction then turn to right and walks 10 m again turn to right
and walks 36 m. Poonam starts from point P and walks 5 m in south direction and then he turns his right and walk 25 m and again he turns his left and walks 20 m.
Prateek starts from point A walks 28 m east then turn to right and walks 21 m.

hi Passage

िन िलिखत जानकारी क ा यानपू व क अ ययन करे औ र नीचे द ए ग ए के उ र दीिजये:

इस तरह से PQ अ ह P उ र म है और Q दि ण दशा म है। इस तरह से AB अ है, A पि म दशा म है और B पूव दशा म है। PQ अ और AB अ एक बंदु Z पर इस
तरह से ित छेद करते ह क PZ 25 मीटर है, ZQ 21 मीटर है, ZA 19 मीटर है, ZB 18 मीटर है। अतुल बंदु Q से चलना शु करता है और दि ण दशा म 20 मीटर चलता है
और फर वह दाय ओर मुड़ता है और 29 मीटर चलता है। रा ल बंदु B से चलना शु करता है और उ र दशा म 15 मीटर चलता है फर दाएं मुड़ता है और 10 मीटर चलता है
फर से दाय मुड़ता है और 36 मीटर चलता है। पूनम बंदु P से चलना शु करती है और दि ण दशा म 5 मीटर चलती है और फर वह अपने दाय मुड़ती है और 25 मीटर चलती
है और फर से वह बाय ओर मुड़ती है और 20 मीटर चलती है। तीक बंदु A से चलना शु करता है 28 मीटर पूव क ओर जाता है फर दाय ओर मुड़ता है और 21 मीटर चलता
है।

Q­33 What is distance between Poonam’s current position and B?
eng

01. 43 m 02. 45 m


03. 33 m 04. 42 m
05. 47m
Q­33 पूनम क वतमान थान और B के बीच क दूरी या है?
hi

01. 43 मी 02. 45 मी


03. 33 मी 04. 42 मी
05. 47 मी
Solution for Question 33 Ans Key: 1

eng

hi

Q­34 Point B is in which direction with respect to Atul’s current position?
eng

01. North 02. East


03. North­east 04. North­west
05. South
Q­34 बंदु B, अतुल क वतमान थान के संदभ म कस दशा म है?
hi

01. उ र 02. पूव


03. उ र­पूव 04. उ र­पि म
05. दि ण
Solution for Question 34 Ans Key: 3

eng

hi

Q­35 Prateek’s current position is in which direction with respect to Rahul’s current position?
eng

01. South 02. South­east


03. South­west 04. West
05. North­west
Q­35 तीक का वतमान थान रा ल के वतमान थान के स दभ म कस दशा म है?
hi

01. दि ण 02. दि ण­पूव


03. दि ण­पि म 04. पि म
05. उ र­पि म
Solution for Question 35 Ans Key: 4

eng

hi
Q­36 Rahul’s current position how far and in which direction with respect to Atul’s first turn? (Approx.) 
eng

01. 43 m, North 02. 45 m, East


03. 34 m, North­east 04. 42 m, North­east
05. 35 m, North­west
Q­36 अतुल क पहली मोड़ के स दभ म रा ल का वतमान थान कतनी दूरी और कस दशा म है? (लगभग)
hi

01. 43 मीटर, उ र 02. 45 मीटर, पूव


03. 34 मीटर, उ र­पूव 04. 42 मीटर, उ र­पूव
05. 35 मीटर, उ र­पि म
Solution for Question 36 Ans Key: 3
eng

hi

Q­37 Each of the questions below consists of a question and two statements numbered I and II given below it. You have to decide whether the data
eng provided in the statements are sufficient to answer the question. Read both the statements and Give answer:

Eight friends named A, B, C, D, E, F, G and H are sitting around a rectangle table facing the centre. B is sitting opposite to whom?
I. A is an immediate neighbor of B and D. C is third to the right of B. H is exactly between G and F. G is not an immediate neighbour of E. 
II. C is sitting third to right of A. D is sitting immediate right of C and is an immediate neighbour of B. G is sitting immediate left of C. There is one person
between E and H.

01. If the data in Statement I alone are sufficient 02. If the data in Statement II alone are sufficient to answer the question, while the data in Statement I


to answer the question, while the data in alone are not sufficient to answer the question.
Statement II alone are not sufficient to
answer the question.
03. If the data in Statement I alone or in 04. If the data in both the Statements I and II are not sufficient to answer the question.
Statement II alone are sufficient to answer
the question.
05. If the data in both the Statements I and II
together are necessary to answer the
question.
Q­37 नीचे द ए ग ए ये क म एक औ र उसके नीचे द ो कथन I औ र II  द ए ग ए ह । आपको य ह त य करना है क कथन म दया गया डाटा का उ र
hi दे न े के ि ल ए पया है य ा नह है । दोन कथन क ो प ढ़ए औ र उ र दीिजए

A, B, C, D, E, F, G और H आठ िम क क ओर मुख करके एक आयताकार मेज के चार ओर बैठे ह। B  कसके िवपरीत बैठा है?
I. A, B और D का तुरत ं पड़ोसी है। C, B के दाय से तीसरा है। H, G और F के ठीक बीच म है। G, E का तुरत ं पड़ोसी नह है।
II. C, A के दाएं से तीसरे थान पर बैठा है। D, C के तुरत
ं दाएं बैठा है और B का तुरत
ं पड़ोसी है। G, C के तुरत
ं बाएं बैठा है। E और H के बीच म एक ि है।

01. केवल कथन I म दया गया डाटा का उ र 02. य द केवल कथन II म दया गया डाटा का उ र देने के िलए पया है जब क केवल कथन I म दया
देने के िलए पया है जब क केवल कथन II म गया डाटा का उ र देने के िलए पया नह है।
दया गया डाटा का उ र देने के िलए
पया नह है।
03. य द या तो केवल कथन I या केवल कथन II म 04. य द कथन I और कथन II दोन का डाटा िमलकर भी का उ र देने के िलए पया नह है।
दया गया डाटा का उ र देने के िलए
पया है।
05. य द दोन कथन I व II एक साथ उ र देने के
िलये पया ह ले कन अकेले कोई भी कथन
पया नह है।
Solution for Question 37 Ans Key: 4
eng

hi

Q­38 Each of the questions below consists of a question and two statements numbered I and II given below it. You have to decide whether the data
eng provided in the statements are sufficient to answer the question. Read both the statements and Give answer:

The eight persons P, Q, R, S, T, U, W and V were born in different months from January to August not necessarily in the same order. Who among the
following was born in July? 
I. Q was born in a month which was having 31 days. Three persons were born between Q and P. P was born after Q.  
II. U was born just after P. R was born in a month which was having least number of days. S was born immediate after U.

01. If the data in Statement I alone are sufficient 02. If the data in Statement II alone are sufficient to answer the question, while the data in Statement I


to answer the question, while the data in alone are not sufficient to answer the question.
Statement II alone are not sufficient to
answer the question.
03. If the data in Statement I alone or in 04. If the data in both the Statements I and II are not sufficient to answer the question.
Statement II alone are sufficient to answer
the question.
05. If the data in both the Statements I and II
together are necessary to answer the
question.
Q­38 नीचे द ए ग ए ये क म एक औ र उसके नीचे द ो कथन I औ र II  द ए ग ए ह । आपको य ह त य करना है क कथन म दया गया डाटा का उ र
hi दे न े के ि ल ए पया है य ा नह है । दोन कथन क ो प ढ़ए औ र उ र दीिजए

आठ ि P, Q, R, S, T, U, W और V का ज म अलग­अलग महीन जनवरी से अग त तक म आ था ले कन ज री नह क इसी म म हो। िन िलिखत म से कौन


जुलाई म पैदा आ था?
I. Q का ज म उस महीने म आ था, िजसम 31  दन थे। Q और P के बीच म तीन ि य का ज म आ था। Q के बाद P का ज म आ था।
II. P के ज म के ठीक बाद U का ज म आ था। R का ज म उस महीने म आ था िजसम कम से कम दन थे। S का ज म, U के तुरत
ं बाद आ था।

01. केवल कथन I म दया गया डाटा का उ र 02. य द केवल कथन II म दया गया डाटा का उ र देने के िलए पया है जब क केवल कथन I म दया
देने के िलए पया है जब क केवल कथन II म गया डाटा का उ र देने के िलए पया नह है।
दया गया डाटा का उ र देने के िलए
पया नह है।
03. य द या तो केवल कथन I या केवल कथन II म 04. य द कथन I और कथन II दोन का डाटा िमलकर भी का उ र देने के िलए पया नह है।
दया गया डाटा का उ र देने के िलए
पया है।
05. य द दोन कथन I व II एक साथ उ र देने के
िलये पया ह ले कन अकेले कोई भी कथन
पया नह है।
Solution for Question 38 Ans Key: 5

eng January February March April May June July August


Q R P U S
hi जनवर फरवर माच अ ैल मई जून जुल ाई अग त

Q R P U S

Q­39 Each of the questions below consists of a question and two statements numbered I and II given below it. You have to decide whether the data
eng provided in the statements are sufficient to answer the question.Read both the statements and Give answer:

Is A is grandmother of C, among the six family members A, B, C, D, E and F? 
I. A is the mother­in­law of D, E is grandson of F, who is the father of B. 
II. B is the father of C, who is grandson of F. D is the wife of F’s son and C’s father.

01. If the data in Statement I alone are sufficient 02. If the data in Statement II alone are sufficient to answer the question, while the data in Statement I


to answer the question, while the data in alone are not sufficient to answer the question.
Statement II alone are not sufficient to
answer the question.
03. If the data in Statement I alone or in 04. If the data in both the Statements I and II are not sufficient to answer the question.
Statement II alone are sufficient to answer
the question.
05. If the data in both the Statements I and II
together are necessary to answer the
question.
Q­39 नीचे द ए ग ए ये क म एक औ र उसके नीचे द ो कथन I औ र II  द ए ग ए ह । आपको य ह त य करना है क कथन म दया गया डाटा का उ र
hi दे न े के ि ल ए पया है य ा नह है । दोन कथन क ो प ढ़ए औ र उ र दीिजए

प रवार के छह सद य A, B, C, D, E और F के बीच म या A, C क डमदर है?


I. A, D क सास है, E, F का डसन है, जो B का िपता है।
II. B, C का िपता है, जो F का डसन है। D, F के बेटे और C के िपता क प ी है।

01. केवल कथन I म दया गया डाटा का उ र 02. य द केवल कथन II म दया गया डाटा का उ र देने के िलए पया है जब क केवल कथन I म दया
देने के िलए पया है जब क केवल कथन II म गया डाटा का उ र देने के िलए पया नह है।
दया गया डाटा का उ र देने के िलए
पया नह है।
03. य द या तो केवल कथन I या केवल कथन II म 04. य द कथन I और कथन II दोन का डाटा िमलकर भी का उ र देने के िलए पया नह है।
दया गया डाटा का उ र देने के िलए
पया है।
05. य द दोन कथन I व II एक साथ उ र देने के
िलये पया ह ले कन अकेले कोई भी कथन
पया नह है।
Solution for Question 39 Ans Key: 5
eng
hi

Q­40 Study the following alphanumeric series carefully and answer the questions given below:
eng
68EAFR    73WBO     52ENDO     84TVZR     65GRTY
The letters which are attached with the numbers in which at least one is odd digit are arranged according to the descending orders of their numbers from left
to right and then all letters of the words are arranged in the alphabetical order individually, then which of the following is the 9th letter from the left end
among letters? 

01. M 02. O
03. N 04. R
05. None of these
Q­40 िन िलिखत अ फ़ा यू म े रक ृ ं ख ला क ा यानपू व क अ ययन कर औ र नीचे द ए ग ए के उ र द :
hi
68EAFR    73WBO     52ENDO     84TVZR     65GRTY
िजन अ र को सं या के साथ जोड़ा जाता है, िजनम कम से कम एक िवषम अंक है, उन सं या को बाएं से दाएं बढ़ते ए म के अनुसार वि थत कया जाता है
और फर श द के सभी अ र को अलग­अलग प से वणानु म म वि थत कया जाता है,  फर अ र म िन िलिखत म से कौन सा अ र बाएं से 9 वां है? 

01. M 02. O
03. N 04. R
05. इनम से कोई नह
Solution for Question 40 Ans Key: 5
eng 52ENDO     65GRTY      73WBO 

DENOGRTYBOW = B
hi 52ENDO     65GRTY      73WBO 

DENOGRTYBOW   = B

Q­41 Study the following alphanumeric series carefully and answer the questions given below:
eng
68EAFR    73WBO     52ENDO     84TVZR     65GRTY
If all the numbers are arrange in ascending order, then which letter is 10th from the right end among letters?

01. T 02. Y
03. E 04. A
05. O
Q­41 िन िलिखत अ फ़ा यू म े रक ृ ं ख ला क ा यानपू व क अ ययन कर औ र नीचे द ए ग ए के उ र द :
hi
68EAFR    73WBO     52ENDO     84TVZR     65GRTY
य द सभी नंबर को आरोही म म वि थत कया जाता है, तो अ र म दाय छोर से 10 वां अ र कौन सा है?

01. T 02. Y
03. E 04. A
05. O
Solution for Question 41 Ans Key: 4
eng 52ENDO  65GRTY    68EAFR   73WBO   84TVZR 

ENDO    GRTY    EAFR    WBO   TVZR
hi 52ENDO  65GRTY    68EAFR   73WBO   84TVZR 

ENDO    GRTY    EAFR    WBO   TVZR

Study the following alphanumeric series carefully and answer the questions given below:
Q­42 Study the following alphanumeric series carefully and answer the questions given below:
eng
68EAFR    73WBO     52ENDO     84TVZR     65GRTY
The words are arranged according to the ascending order of the numbers which are attached to them from left to right, then what is sum of the resultant of
the sum of the number and the position of letters of the series which is third from the left and second from the right?

01. 205 02. 202


03. 227 04. 210
05. 211
Q­42 िन िलिखत अ फ़ा यू म े रक ृ ं ख ला क ा यानपू व क अ ययन कर औ र नीचे द ए ग ए के उ र द :
hi
68EAFR    73WBO     52ENDO     84TVZR     65GRTY
श द को सं या के आरोही म के अनुसार बाएं से दाएं वि थत कया जाता है जो उनके बाय से जुड़े ह,  फर बाय से तीसरी और दाय से दूसरी, सं या और अ र
के थान के योग के योग का प रणाम या है?

01. 205 02. 202


03. 227 04. 210
05. 211
Solution for Question 42 Ans Key: 5
eng 52ENDO       65GRTY        68EAFR    73WBO    84TVZR   

68EAFR = 68+5+1+6+18= 98

73WBO = 73+23+2+15= 113,   98+113= 211
hi 52ENDO       65GRTY        68EAFR    73WBO    84TVZR   

68EAFR = 68+5+1+6+18= 98

73WBO = 73+23+2+15= 113,   98+113= 211

Q­43 Study the following alphanumeric series carefully and answer the questions given below:
eng
68EAFR    73WBO     52ENDO     84TVZR     65GRTY
The words are arranged according to the descending order of the numbers which are attached to them from left to right and all series (letters which are
attached with the numbers) are merge, then how many odd number are there in the given series each of which is immediately preceded by number and
immediately followed by vowel?

01. None 02. Two


03. Three 04. Four
05. More than four
Q­43 िन िलिखत अ फ़ा यू म े रक ृ ं ख ला क ा यानपू व क अ ययन कर औ र नीचे द ए ग ए के उ र द :
hi
68EAFR    73WBO     52ENDO     84TVZR     65GRTY
श द को उनक सं या के आधार पर बाय से दाय अवरोही म के अनुसार वि थत कया जाता है और सभी ृंखला  (िजसम अ र को सं या के साथ जोड़ा
गया है) िमला दया जाता है,  फर दी गई ृंखला म कतनी िवषम सं या है िजनम से येक के तुरत
ं पहले सं या और तुरत
ं बाद म वर है?

01. कोई नह 02. दो


03. तीन 04. चार
05. चार से अिधक
Solution for Question 43 Ans Key: 1
eng 84TVZR73WBO68EAFR65GRTY52ENDO       
hi 84TVZR73WBO68EAFR65GRTY52ENDO       

Q­44 Study the following alphanumeric series carefully and answer the questions given below:
eng
68EAFR    73WBO     52ENDO     84TVZR     65GRTY
How many meaningful words can be formed from the letters of the words which are attached with the numbers in which at least one digit is odd in the given
alphanumeric series? 

01. One 02. Two


03. Three 04. Four
05. More than four
Q­44 िन िलिखत अ फ़ा यू म े रक ृ ं ख ला क ा यानपू व क अ ययन कर औ र नीचे द ए ग ए के उ र द :
hi
68EAFR    73WBO     52ENDO     84TVZR     65GRTY
उन श द के अ र से कतने साथक श द बनाए जा सकते ह िजनम दी गई अ रांक य ृंखला म श द के साथ जो सं याय लगी ह उनम कम से कम एक अंक िवषम
सं या है?  

01. एक 02. दो
03. तीन 04. चार
05. चार से अिधक
Solution for Question 44 Ans Key: 3
eng WOB, BOW, DONE
hi WOB, BOW, DONE

Q­45 Each of the questions below consists of a question and two statements numbered I and II given below it. You have to decide whether the data
eng provided in the statements are sufficient to answer the question.Read both the statements and Give answer:

Seven Books Chemistry, Physics, Maths, Biology, Social, English and Hindi are to be stacked together. Bottom book is given number 1, then 2 and so on.
Which is given number 3? 
I. There are only three book between Biology and Maths. Chemistry is giving even number place. English is placed immediate below to Biology. English is
not given even number place.
II. Maths is given even number place. Hindi and Social is not given in top place and bottom place. Social is given odd number place. There are 1 books
placed between Physics and Social.

01. If the data in Statement I alone are sufficient 02. If the data in Statement II alone are sufficient to answer the question, while the data in Statement I


to answer the question, while the data in alone are not sufficient to answer the question.
Statement II alone are not sufficient to
answer the question.
03. If the data in Statement I alone or in 04. If the data in both the Statements I and II are not sufficient to answer the question.
Statement II alone are sufficient to answer
the question.
05. If the data in both the Statements I and II
together are necessary to answer the
question.
Q­45 नीचे द ए ग ए ये क म एक औ र उसके नीचे द ो कथन I औ र II  द ए ग ए ह । आपको य ह त य करना है क कथन म दया गया डाटा का उ र
hi दे न े के ि ल ए पया है य ा नह है । दोन कथन क ो प ढ़ए औ र उ र दीिजए

सात कताब केिम ी, भौितक , गिणत, बायोलॉजी, सोशल, अं ज े ी और हंदी को एक साथ रखा जाता है। सबसे नीचे क बुक को सं या 1,  फर 2 और इसी तरह से आगे
भी। कौन सी कताब सं या 3 पर है?
I. बायोलॉजी और गिणत के बीच केवल तीन पु तक है। केिम ी सम सं या थान पर है। अं ज
े ी को बायोलॉजी के तुरत
ं बगल म नीचे रखा गया है। अं ज
े ी को सम सं या
थान पर नह रखा गया है।
II. गिणत सम सं या थान पर रखा गया है। हंदी और सोशल को शीष थान और नीचे थान पर नह रखा जाता है। सोशल को िवषम सं या थान पर रखा जाता है।
भौितक और सोशल के बीच म एक पु तक रखी गई ह।

01. केवल कथन I म दया गया डाटा का उ र 02. य द केवल कथन II म दया गया डाटा का उ र देने के िलए पया है जब क केवल कथन I म दया
देने के िलए पया है जब क केवल कथन II म गया डाटा का उ र देने के िलए पया नह है।
दया गया डाटा का उ र देने के िलए
पया नह है।
03. य द या तो केवल कथन I या केवल कथन II म 04. य द कथन I और कथन II दोन का डाटा िमलकर भी का उ र देने के िलए पया नह है।
दया गया डाटा का उ र देने के िलए
पया है।
05. य द दोन कथन I व II एक साथ उ र देने के
िलये पया ह ले कन अकेले कोई भी कथन
पया नह है।
Solution for Question 45 Ans Key: 5
eng Physics

Maths

Social

Chemistry

Hindi

Biology
English
hi भौितक

गिणत

सोशल

केिम ी

हंदी

बायोलॉजी

अं ज
े ी

Q­46 In the following question, three short sentences are given. Choose the meaningful sentence which has been formed after combining the all
eng three short sentences.

A. The Reserve Bank of India will release.
B. Third bi­monthly monetary policy statement for 2020­21 on October 9.
C. October 1 as originally envisaged.

01. The Reserve Bank of India will release their 02. The Reserve Bank of India will release its third bi­monthly monetary policy statement for 2020­21


third bi­monthly monetary policy statement on October 9, instead of October 1 as originally envisaged.
for 2020­21 on October 9, instead of
October 1 as originally envisaged.
03. The Reserve Bank of India will release its 04. The Reserve Bank of India will release its third bi­monthly monetary policy statement for 2020­21
third bi­monthly monetary policy statement on October 9, despite of October 1 as originally envisaged.
for 2020­21 on October 9, inspite of
October 1 as originally envisaged.
05. None of the above.
Q­46 In the following question, three short sentences are given. Choose the meaningful sentence which has been formed after combining the all
hi three short sentences.

A. The Reserve Bank of India will release.
B. Third bi­monthly monetary policy statement for 2020­21 on October 9.
C. October 1 as originally envisaged.

01. The Reserve Bank of India will release their 02. The Reserve Bank of India will release its third bi­monthly monetary policy statement for 2020­21


third bi­monthly monetary policy statement on October 9, instead of October 1 as originally envisaged
for 2020­21 on October 9, instead of
October 1 as originally envisaged.
03. The Reserve Bank of India will release its 04. The Reserve Bank of India will release its third bi­monthly monetary policy statement for 2020­21
third bi­monthly monetary policy statement on October 9, despite of October 1 as originally envisaged.
for 2020­21 on October 9, inspite of
October 1 as originally envisaged.
05. None of the above.
Solution for Question 46 Ans Key: 2
eng The Reserve Bank of India will release its (used for singular subject) third bi­monthly monetary policy statement for 2020­21 on October 9, instead of (in
place of) October 1 as originally envisaged.
hi The Reserve Bank of India will release its (used for singular subject) third bi­monthly monetary policy statement for 2020­21 on October 9, instead of (in
place of) October 1 as originally envisaged.

Q­47 In the following question, three short sentences are given. Choose the meaningful sentence which has been formed after combining the all
eng three short sentences.

A. Indian army is mainly responsible to protect the country.
B. Land based attacks are the main problem.
C. It helps deal with terrorism too.

01. Indian army is mainly responsible to protect 02. Indian army is mainly responsible to protect the country against land based attacks despite it lends a


the country for land based attacks and helping hand in dealing with terrorism.
moreover it lends a helping hand in dealing
with terrorism.
03. Indian army is mainly responsible to protect 04. Indian army is mainly responsible to protect the country with land based attacks and it also lends a
the country against land based attacks and it helping hand in dealing with terrorism.
also lends a helping hand in dealing with
also lends a helping hand in dealing with
terrorism.
05. None of the above
Q­47 In the following question, three short sentences are given. Choose the meaningful sentence which has been formed after combining the all
hi three short sentences.

A. Indian army is mainly responsible to protect the country.
B. Land based attacks are the main problem.
C. It helps deal with terrorism too.

01. Indian army is mainly responsible to protect 02. Indian army is mainly responsible to protect the country against land based attacks despite it lends a


the country for land based attacks and helping hand in dealing with terrorism
moreover it lends a helping hand in dealing
with terrorism.
03. Indian army is mainly responsible to protect 04. Indian army is mainly responsible to protect the country with land based attacks and it also lends a
the country against land based attacks and it helping hand in dealing with terrorism
also lends a helping hand in dealing with
terrorism
05. None of the above
Solution for Question 47 Ans Key: 1
eng For represents the ‘favour’ and a soldier acts for the welfare of the country. Therefore, option (1) should be the correct answer.
hi For represents the ‘favour’ and a soldier acts for the welfare of the country. Therefore, option (1) should be the correct answer.

Q­48 In the following question, three short sentences are given. Choose the meaningful sentence which has been formed after combining the all
eng three short sentences.

A. It is natural for spiritual aspirant who has undertaken a study.
B. The scriptures to evaluate the different means.
C. Liberations recommended by them.

01. It is natural for a spiritual aspirant which has 02. It is natural for a spiritual aspirant who has undertaken a study of the scriptures, to evaluate the


undertaken a study of the scriptures, to different means of liberation recommends by them.
evaluating the different means to liberation
recommended by them.
03. It is natural for a spiritual aspirant whom has 04. It is natural for a spiritual aspirant who has undertaken a study of the scriptures, to evaluate the
undertaken a study of the scriptures, to different means to liberation recommended by them.
evaluating the different means to liberation
recommended to them.
05. None of the above.
Q­48 In the following question, three short sentences are given. Choose the meaningful sentence which has been formed after combining the all
hi three short sentences.

A. It is natural for spiritual aspirant who has undertaken a study.
B. The scriptures to evaluate the different means.
C. Liberations recommended by them.

01. It is natural for a spiritual aspirant which has 02. It is natural for a spiritual aspirant who has undertaken a study of the scriptures, to evaluate the


undertaken a study of the scriptures, to different means of liberation recommends by them.
evaluating the different means to liberation
recommended by them.
03. It is natural for a spiritual aspirant whom has 04. It is natural for a spiritual aspirant who has undertaken a study of the scriptures, to evaluate the
undertaken a study of the scriptures, to different means to liberation recommended by them
evaluating the different means to liberation
recommended to them.
05. None of the above.
Solution for Question 48 Ans Key: 4
eng Option (4) is the correct answer.
It is natural for a spiritual aspirant who (relative pronoun) has undertaken a study of the scriptures, to evaluate (to + V1) the different means to liberation
recommended by them.
hi Option (4) is the correct answer.
It is natural for a spiritual aspirant who (relative pronoun) has undertaken a study of the scriptures, to evaluate (to + V1) the different means to liberation
recommended by them.

Q­49 In the following question, three short sentences are given. Choose the meaningful sentence which has been formed after combining the all
eng three short sentences.

A. In India, 51 per cent of total sown areas are rain fed.
B. these areas contribute nearly 40 per cent of the total food production in the country,
C. the Union Ministry of Agriculture and Farmers’ Welfare.

01. In India, 51 per cent of total sown areas are 02. In India, 51 per cent of total sown areas are rain fed but these areas contribute nearly 40 per cent


rain fed while these areas contribute nearly of the total food production in the country, by the Union Ministry of Agriculture and Farmers’
40 per cent of the total food production in Welfare.
the country, according to the Union Ministry
of Agriculture and Farmers’ Welfare.
03. In India, 51 per cent of total sown areas are 04. In India, 51 per cent of total sown areas are rain fed however, these areas contribute nearly 40 per
rain fed and these areas contribute nearly 40 cent of the total food production in the country, according to the Union Ministry of Agriculture and
per cent of the total food production in the Farmers’ Welfare.
country, according to the Union Ministry of
Agriculture and Farmers’ Welfare.
05. None of the above.
Q­49 In the following question, three short sentences are given. Choose the meaningful sentence which has been formed after combining the all
hi three short sentences.

A. In India, 51 per cent of total sown areas are rain fed.
B. these areas contribute nearly 40 per cent of the total food production in the country,
C. the Union Ministry of Agriculture and Farmers’ Welfare.

01. In India, 51 per cent of total sown areas are 02. In India, 51 per cent of total sown areas are rain fed but these areas contribute nearly 40 per cent


rain fed while these areas contribute nearly of the total food production in the country, by the Union Ministry of Agriculture and Farmers’
40 per cent of the total food production in Welfare.
the country, according to the Union Ministry
of Agriculture and Farmers’ Welfare.
03. In India, 51 per cent of total sown areas are 04. In India, 51 per cent of total sown areas are rain fed however, these areas contribute nearly 40 per
rain fed and these areas contribute nearly 40 cent of the total food production in the country, according to the Union Ministry of Agriculture and
per cent of the total food production in the Farmers’ Welfare.
country, according to the Union Ministry of
Agriculture and Farmers’ Welfare
05. None of the above.
Solution for Question 49 Ans Key: 3
eng Option (3) is the correct answer.
‘And’ should be the perfect connector because it combines the similar structure.
hi Option (3) is the correct answer.
‘And’ should be the perfect connector because it combines the similar structure.

Q­50 In the following question, three short sentences are given. Choose the meaningful sentence which has been formed after combining the all
eng three short sentences.

A. The decision to convert government employees’ leave travel concession.
B. A reimbursable expenditure allowance.
C. The civil servant supplements the amount with cash of his own, is a smart move

01. The decision to convert government 02. The decision to convert government employee’s leave travel concession into a reimbursable


employees’ leave travel concession by a expenditure allowance, provides the civil servant supplements the amount with cash of his own, is a
reimbursable expenditure allowance, smart move.
provides the civil servant supplements the
amount with cash of his own, is a smart
move.
03. The decision to convert government 04. The decision to convert government employees’ leave travel concession into a reimbursable
employees’ leave travel concession from a expenditure allowance, provided the civil servant supplements the amount with cash of his own, is a
reimbursable expenditure allowance, smart move.
provided the civil servant supplements the
amount with cash of his own, is a smart
move.
05. None of the above.
Q­50 In the following question, three short sentences are given. Choose the meaningful sentence which has been formed after combining the all
hi three short sentences.

A. The decision to convert government employees’ leave travel concession.
B. A reimbursable expenditure allowance.
C. The civil servant supplements the amount with cash of his own, is a smart move

01. The decision to convert government 02. The decision to convert government employee’s leave travel concession into a reimbursable


employees’ leave travel concession by a expenditure allowance, provides the civil servant supplements the amount with cash of his own, is a
reimbursable expenditure allowance, smart move.
provides the civil servant supplements the
amount with cash of his own, is a smart
move.
03. The decision to convert government 04. The decision to convert government employees’ leave travel concession into a reimbursable
employees’ leave travel concession from a expenditure allowance, provided the civil servant supplements the amount with cash of his own, is a
reimbursable expenditure allowance, smart move
provided the civil servant supplements the
amount with cash of his own, is a smart
move.
05. None of the above.
Solution for Question 50 Ans Key: 4
eng Option (4) is the correct answer.
The decision to convert government employees’ (apstrophe after a word shows both plural and possession) leave travel concession into (expressing a
change of state) a reimbursable expenditure allowance, provided the civil servant supplements the amount with cash of his (shows association with a male
person/animal previously mentioned) own, is a smart move.
hi Option (4) is the correct answer.
The decision to convert government employees’ (apstrophe after a word shows both plural and possession) leave travel concession into (expressing a
change of state) a reimbursable expenditure allowance, provided the civil servant supplements the amount with cash of his (shows association with a male
person/animal previously mentioned) own, is a smart move.

Q­51 In the following question, a long sentence with a highlighted phrase is written five times. You have to choose the sentence in which the
eng highlighted phrase is grammatically correct.

01. Leaving aside little room for 02. Leaving less little room for misinterpretation, the senior politician offered clarification about his


misinterpretation, the senior politician role in the past election.
offered clarification about his role in the past
election.
03. Leaving little room for misinterpretation, 04. Having left little room for misinterpretation, the senior politician offered clarification about his role
the senior politician offered clarification in the past election.
about his role in the past election.
05. Leaving aside a little room for
misinterpretation, the senior politician
offered clarification about his role in the past
election.
Q­51 In the following question, a long sentence with a highlighted phrase is written five times. You have to choose the sentence in which the
hi highlighted phrase is grammatically correct.

01. Leaving aside little room for 02. Leaving less little room for misinterpretation, the senior politician offered clarification about his


misinterpretation, the senior politician role in the past election.
offered clarification about his role in the past
election.
03. Leaving little room for misinterpretation, 04. Having left little room for misinterpretation, the senior politician offered clarification about his role
the senior politician offered clarification in the past election.
about his role in the past election
05. Leaving aside a little room for
misinterpretation, the senior politician
offered clarification about his role in the past
election.
Solution for Question 51 Ans Key: 3
eng Option (3) is the correct answer.
If you say there is ‘no/little room for a feeling or type of behaviour’, you mean ‘it is not acceptable’.
hi Option (3) is the correct answer.
If you say there is ‘no/little room for a feeling or type of behaviour’, you mean ‘it is not acceptable’.

Q­52 In the following question, a long sentence with a highlighted phrase is written five times. You have to choose the sentence in which the
eng highlighted phrase is grammatically correct.

01. The world’s population will continue to 02. The world’s population will continue to grow as long as the birthrate exceeds the death­rate.


01. The world’s population will continue to 02. The world’s population will continue to grow as long as the birthrate exceeds the death­rate.
grow when the birthrate exceeds the death­
rate.
03. The world’s population will continue to 04. The world’s population will continue to grow until after the birthrate exceeds the death­rate.
grow unless the birthrate exceeds the
death­rate.
05. The world’s population will continue to
grow and the birthrate exceeds the death­
rate.
Q­52 In the following question, a long sentence with a highlighted phrase is written five times. You have to choose the sentence in which the
hi highlighted phrase is grammatically correct.

01. The world’s population will continue to 02. The world’s population will continue to grow as long as the birthrate exceeds the death­rate.


grow when the birthrate exceeds the death­
rate
03. The world’s population will continue to 04. The world’s population will continue to grow until after the birthrate exceeds the death­rate
grow unless the birthrate exceeds the
death­rate
05. The world’s population will continue to
grow and the birthrate exceeds the death­
rate
Solution for Question 52 Ans Key: 2
eng ‘as long as’ supports the statement because ‘it describes the simultaneous duration between the two events’.
Option (2) is the correct answer.
hi ‘as long as’ supports the statement because ‘it describes the simultaneous duration between the two events’.
Option (2) is the correct answer.

Q­53 In the following question, a long sentence with a highlighted phrase is written five times. You have to choose the sentence in which the
eng highlighted phrase is grammatically correct.

01. All the members of the club were 02. All the members of the club had assembled to celebrate the 50th anniversary of the club.


assembled to celebrate the 50th
anniversary of the club.
03. All the members of the club were 04. All the members of the club had been assembled to celebrate the 50th anniversary of the club.
assembling to celebrate the 50th
anniversary of the club.
05. All the members of the club assembled to
celebrate the 50th anniversary of the club.
Q­53 In the following question, a long sentence with a highlighted phrase is written five times. You have to choose the sentence in which the
hi highlighted phrase is grammatically correct.

01. All the members of the club were 02. All the members of the club had assembled to celebrate the 50th anniversary of the club.


assembled to celebrate the 50th
anniversary of the club.
03. All the members of the club were 04. All the members of the club had been assembled to celebrate the 50th anniversary of the club.
assembling to celebrate the 50th
anniversary of the club.
05. All the members of the club assembled to
celebrate the 50th anniversary of the club
Solution for Question 53 Ans Key: 5
eng Since, the sentence is about the past event Option (5) is the correct answer.
hi Since, the sentence is about the past event Option (5) is the correct answer.

Q­54 In the following question, a long sentence with a highlighted phrase is written five times. You have to choose the sentence in which the
eng highlighted phrase is grammatically correct.

01. The leader didn’t accede at the demand 02. The leader wasn’t accede at the demand of the people as he was aware of the risk involved in it.


of the people as he was aware of the risk
involved in it.
03. The leader wasn’t acceded of the 04. The leader didn’t accede to the demand of the people as he was aware of the risk involved in it.
demand of the people as he was aware of
the risk involved in it.
the risk involved in it.
05. The leader didn’t accede by the demand
of the people as he was aware of the risk
involved in it.
Q­54 In the following question, a long sentence with a highlighted phrase is written five times. You have to choose the sentence in which the
hi highlighted phrase is grammatically correct.

01. The leader didn’t accede at the demand 02. The leader wasn’t accede at the demand of the people as he was aware of the risk involved in it.


of the people as he was aware of the risk
involved in it.
03. The leader wasn’t acceded of the 04. The leader didn’t accede to the demand of the people as he was aware of the risk involved in it
demand of the people as he was aware of
the risk involved in it.
05. The leader didn’t accede by the demand
of the people as he was aware of the risk
involved in it.
Solution for Question 54 Ans Key: 4
eng The word ‘accede’ is generally followed by ‘to’. 
Option (4) is the correct answer.
hi The word ‘accede’ is generally followed by ‘to’. 
Option (4) is the correct answer.

Q­55 In the following question, a long sentence with a highlighted phrase is written five times. You have to choose the sentence in which the
eng highlighted phrase is grammatically correct.

01. He has been receiving no other message 02. He has been receiving no other message than an urgent telegram asking him rushing at his


than an urgent telegram asking him to rush village immediately.
his village immediately.
03. He has been receiving no other message 04. He has been receiving no other message than an urgent telegram asking him to rush to his village
than an urgent telegram asked him to rush immediately.
his village immediately.
05. He has been receiving no other message
than an urgent telegram asking him to rush
for his village immediately.
Q­55 In the following question, a long sentence with a highlighted phrase is written five times. You have to choose the sentence in which the
hi highlighted phrase is grammatically correct.

01. He has been receiving no other message 02. He has been receiving no other message than an urgent telegram asking him rushing at his


than an urgent telegram asking him to rush village immediately.
his village immediately.
03. He has been receiving no other message 04. He has been receiving no other message than an urgent telegram asking him to rush to his village
than an urgent telegram asked him to rush immediately
his village immediately.
05. He has been receiving no other message
than an urgent telegram asking him to rush
for his village immediately.
Solution for Question 55 Ans Key: 4
eng Option (4) is the correct answer.
He has been receiving no other message than an urgent telegram asking him to rush (to + V1) to (‘rush’ takes ‘to’ with it) his village immediately.
hi Option (4) is the correct answer.
He has been receiving no other message than an urgent telegram asking him to rush (to + V1) to (‘rush’ takes ‘to’ with it) his village immediately.

Passage for Question 56, 57, 58 (ST­2082 CRP PO MANS (6­10) 2020)
eng Passage

In the following passage, some blanks are given and against each five words are suggested to fill in the blank. You have to choose a pair of words
which can appropriately be filled in the blank.

The Reserve Bank of India’s (RBI’s) response to the covid­induced economic crisis so far is ___________ (A) and well­calibrated. It also resorted to several
“unconventional" measures to address market dislocations. Financial conditions eased across segments with front­loaded cuts in the policy repo rate and large
system­wide and targeted liquidity injections. ___________ (B) of surplus liquidity significantly reduced borrowing costs. While some transmission happened
through bank lending rates, transmission through short­term rates, and three­ to five­year bond yields was more ___________ (C).
Besides the standard tools, other RBI measures like asset purchase schemes, opening up of a special liquidity window for the mutual fund industry, regulatory
__________ (D), loan moratoriums and restructuring of debt significantly eased the pain in the financial sector. Global monetary easing coupled with the central
bank’s timely stabilization measures led to a return of foreign portfolio inflows (mostly in equity segments) after record outflows in March.
Strong capital inflows and a weak dollar led to nearly 4%­plus appreciation of the rupee between April and September despite the central bank’s continuous dollar
buying to build up forex reserves.
However, there are a few concerns. Despite the RBI’s multi­pronged response, credit growth is muted due to concerns over lending risks, and the sovereign bond
market is again on ___________ (E). The gilt players remain wary of high Consumer Price Index (CPI) inflation prints primarily on account of supply­side
disruptions and higher taxes on fuels, and the huge supply of government papers as the central government’s gross market borrowings are __________ (F) at ₹12
trillion.

hi Passage

In the following passage, some blanks are given and against each five words are suggested to fill in the blank. You have to choose a pair of words
which can appropriately be filled in the blank.

The Reserve Bank of India’s (RBI’s) response to the covid­induced economic crisis so far is ___________ (A) and well­calibrated. It also resorted to several
“unconventional" measures to address market dislocations. Financial conditions eased across segments with front­loaded cuts in the policy repo rate and large
system­wide and targeted liquidity injections. ___________ (B) of surplus liquidity significantly reduced borrowing costs. While some transmission happened
through bank lending rates, transmission through short­term rates, and three­ to five­year bond yields was more ___________ (C).
Besides the standard tools, other RBI measures like asset purchase schemes, opening up of a special liquidity window for the mutual fund industry, regulatory
__________ (D), loan moratoriums and restructuring of debt significantly eased the pain in the financial sector. Global monetary easing coupled with the central
bank’s timely stabilization measures led to a return of foreign portfolio inflows (mostly in equity segments) after record outflows in March.
Strong capital inflows and a weak dollar led to nearly 4%­plus appreciation of the rupee between April and September despite the central bank’s continuous dollar
buying to build up forex reserves.
However, there are a few concerns. Despite the RBI’s multi­pronged response, credit growth is muted due to concerns over lending risks, and the sovereign bond
market is again on ___________ (E). The gilt players remain wary of high Consumer Price Index (CPI) inflation prints primarily on account of supply­side
disruptions and higher taxes on fuels, and the huge supply of government papers as the central government’s gross market borrowings are __________ (F) at ₹12
trillion.

Q­56 Choose the correct option for (A) & (B).
eng

01. Both i & Q 02. Both ii & P


03. Both iii & Q 04. Both ii & R
05. Both i & P
Q­56 Choose the correct option for (A) & (B).
hi

01. Both i & Q 02. Both ii  & P


03. Both iii & Q 04. Both ii & R
05. Both i & P
Solution for Question 56 Ans Key: 2
eng Optimum (n)­ most conducive to a favourable outcome; best.
Persistence (n)­ the fact of continuing in an opinion or course of action in spite of difficulty or opposition.
For other options:
Sluggish (adj.)­ slow­moving or inactive.
Captivate (v)­ attract and hold the interest and attention of; charm.
Indolence (n)­ avoidance of activity or exertion; laziness.
Enigma (n)­ a person or thing that is mysterious or difficult to understand.
hi Optimum (n)­ most conducive to a favourable outcome; best.
Persistence (n)­ the fact of continuing in an opinion or course of action in spite of difficulty or opposition.
For other options:
Sluggish (adj.)­ slow­moving or inactive.
Captivate (v)­ attract and hold the interest and attention of; charm.
Indolence (n)­ avoidance of activity or exertion; laziness.
Enigma (n)­ a person or thing that is mysterious or difficult to understand.

Q­57 Choose the correct option for (C) & (D).
Q­57 Choose the correct option for (C) & (D).
eng

01. Both i & Q 02. Both ii & P


03. Both iii & Q 04. Both ii & R
05. Both i & P
Q­57 Choose the correct option for (C) & (D).
hi

01. Both i &  Q 02. Both ii & P


03. Both iii & Q 04. Both ii & R
05. Both i & P
Solution for Question 57 Ans Key: 1
eng Pronounced (v): very noticeable or marked.
Forbearance (n): patient self­control; restraint and tolerance..
For other options:
Vague (n): not clear 
Embark (v): begin
hi Pronounced (v): very noticeable or marked.
Forbearance (n): patient self­control; restraint and tolerance..
For other options:
Vague (n): not clear 
Embark (v): begin

Q­58 Choose the correct option for (E) & (F).
eng

01. Both i & Q 02. Both ii & P


03. Both iii & Q 04. Both ii & R
05. Both i & P
Q­58 Choose the correct option for (E) & (F).
hi

01. Both i & Q 02. Both ii & P


03. Both  iii & Q 04. Both ii & R
05. Both i & P
Solution for Question 58 Ans Key: 3
eng Tenterhook (n): tolerate
Peg (v)‐ fix (a price, rate, or amount) at a particular level.

For other options: 
Captivate (v)‐ attract and hold the interest and attention of; charm.
Shun (v): to ignore
Alleviate (v): relieve
Plush (adj.)‐ richly luxurious and expensive.

hi Tenterhook (n): tolerate
Peg (v)‐ fix (a price, rate, or amount) at a particular level.

For other options: 
Captivate (v)‐ attract and hold the interest and attention of; charm.
Shun (v): to ignore
Alleviate (v): relieve
Plush (adj.)‐ richly luxurious and expensive.

Passage for Question 59, 60, 61, 62, 63, 64, 65, 66, 67, 68 (ST­2082 CRP PO MANS (24­33) 2020)
eng Passage

Read the following passage and answer the questions based on it. Certain words have been given in bold to help you locate the answers.

A news channel in India alleged recently that several private hospitals in the country were “exposed” by a “sting operation” to be levying fees in excess when
COVID­19 patients went to them for care. It is not clear why a “sting operation” was necessary; the high cost of medical care in the top hospitals of the country is
well known. Anyone who has had major surgery or received intensive care in any of the hospitals can testify to that. The debate now is whether such exorbitant
rates are justified during a pandemic such as the one we are in the midst of, or indeed, ever.

Before we address this question, however, an equally important question arises: why do we have so many private hospitals in a poor country such as India. We have
more hospital beds in the private sector than in the public sector. It is estimated that there are 19 lakh hospital beds, 95,000 ICU beds and 48,000 ventilators in
India. Most of these are concentrated in seven States, Uttar Pradesh, Maharashtra, Tamil Nadu, Kerala, Karnataka, Telangana and West Bengal. Except for Tamil
Nadu, Delhi and West Bengal, there are far more beds and ventilators in the private sector than in the public, according to the Center for Disease Dynamics,
Economics & Policy.

The reason for this abundance of private health care is obviously the lack of adequate public health care. This situation has developed due to two main reasons.
Since Independence, India has, quite rightly, focused attention on the larger picture. The priority in a developing country would be the provision of primary care at
the peripheral level, preventive measures, immunisation, maternity and paediatric care as well as dealing with common infections such as tuberculosis. We have done
this well, resulting in impressive improvements in many health­care indices in the last few decades. However, not enough hospital beds and specialised facilities were
provided by the public sector during this time. At the same time, the burgeoning middle class and increasing wealth produced an explosion in the demand for good
quality health care. Private medicine was quick to capitalise on this demand.

The second reason for the dominance of private medicine in India is the lack of adequate investment in public health. The Indian government spends an abysmally
low 1.3% of GDP on public health care, which is woefully inadequate. Allocation has to be at least double this to address some of our pressing needs. Greater
transparency and tighter administration are necessary to ensure that our resources are utilised appropriately. Specialists should be adequately compensated to
obviate their need for private practice.

Private medicine in India is by no means uniform. It is estimated that there are more than one million unqualified medical practitioners, mostly in the rural areas. Most
of them provide basic health care, charging a modest fee. Some may have claims of expertise (often unproven) in alternative systems of medicine such as ayurveda
and homoeopathy. It is not unheard of them to sometimes venture into minor surgery. At the other end of the spectrum are state­of­the­art corporate hospitals, that
are well equipped and well­staffed and which provide excellent service at high cost. These are often set up in metro cities at huge cost and have successfully
engineered a reverse brain drain of many specialists from pursuing lucrative jobs abroad and staying back in or returning to India. Between the two extremes are a
large number of private practitioners and institutions providing a wide range of services of varying quality. Some are run by trusts, charitable organisations and
religious missions, often providing excellent quality at modest costs.

The public health­care system desperately needs higher government spending. Health care cannot be left to private medicine in a developing country, or indeed, in
any country. Health­care spending by the government must be appropriate, based on evidence, and transparent and accountable. Training of doctors and health­
care workers also need to be the responsibility of the government mainly. Costing and auditing of care and procedures need to be done by independent bodies. This
will not only ensure appropriate care at the right cost but also prevent unreasonable demands of suspicious patients and family.

No hospital, business, institution or individual should profiteer from a national calamity such as the COVID­19 pandemic. Hospitals, like any other institution, have a
social responsibility to provide care in times of need. But one should be also aware of the actual costs involved which have to be met. The cost of medical care
often follows the law of diminishing returns; as the treatment gets more sophisticated, further and further increments, although small, cost enormously more. Some of
the drugs used in the care of severely­ill COVID­19 patients may cost more than ₹50,000 a shot, for example, and may provide only a marginally better outcome.

hi Passage

Read the following passage and answer the questions based on it. Certain words have been given in bold to help you locate the answers.

A news channel in India alleged recently that several private hospitals in the country were “exposed” by a “sting operation” to be levying fees in excess when
COVID­19 patients went to them for care. It is not clear why a “sting operation” was necessary; the high cost of medical care in the top hospitals of the country is
well known. Anyone who has had major surgery or received intensive care in any of the hospitals can testify to that. The debate now is whether such exorbitant
rates are justified during a pandemic such as the one we are in the midst of, or indeed, ever.

Before we address this question, however, an equally important question arises: why do we have so many private hospitals in a poor country such as India. We have
more hospital beds in the private sector than in the public sector. It is estimated that there are 19 lakh hospital beds, 95,000 ICU beds and 48,000 ventilators in
India. Most of these are concentrated in seven States, Uttar Pradesh, Maharashtra, Tamil Nadu, Kerala, Karnataka, Telangana and West Bengal. Except for Tamil
Nadu, Delhi and West Bengal, there are far more beds and ventilators in the private sector than in the public, according to the Center for Disease Dynamics,
Economics & Policy.
The reason for this abundance of private health care is obviously the lack of adequate public health care. This situation has developed due to two main reasons.
Since Independence, India has, quite rightly, focused attention on the larger picture. The priority in a developing country would be the provision of primary care at
the peripheral level, preventive measures, immunisation, maternity and paediatric care as well as dealing with common infections such as tuberculosis. We have done
this well, resulting in impressive improvements in many health­care indices in the last few decades. However, not enough hospital beds and specialised facilities were
provided by the public sector during this time. At the same time, the burgeoning middle class and increasing wealth produced an explosion in the demand for good
quality health care. Private medicine was quick to capitalise on this demand.

The second reason for the dominance of private medicine in India is the lack of adequate investment in public health. The Indian government spends an abysmally
low 1.3% of GDP on public health care, which is woefully inadequate. Allocation has to be at least double this to address some of our pressing needs. Greater
transparency and tighter administration are necessary to ensure that our resources are utilised appropriately. Specialists should be adequately compensated to
obviate their need for private practice.

Private medicine in India is by no means uniform. It is estimated that there are more than one million unqualified medical practitioners, mostly in the rural areas. Most
of them provide basic health care, charging a modest fee. Some may have claims of expertise (often unproven) in alternative systems of medicine such as ayurveda
and homoeopathy. It is not unheard of them to sometimes venture into minor surgery. At the other end of the spectrum are state­of­the­art corporate hospitals, that
are well equipped and well­staffed and which provide excellent service at high cost. These are often set up in metro cities at huge cost and have successfully
engineered a reverse brain drain of many specialists from pursuing lucrative jobs abroad and staying back in or returning to India. Between the two extremes are a
large number of private practitioners and institutions providing a wide range of services of varying quality. Some are run by trusts, charitable organisations and
religious missions, often providing excellent quality at modest costs.

The public health­care system desperately needs higher government spending. Health care cannot be left to private medicine in a developing country, or indeed, in
any country. Health­care spending by the government must be appropriate, based on evidence, and transparent and accountable. Training of doctors and health­
care workers also need to be the responsibility of the government mainly. Costing and auditing of care and procedures need to be done by independent bodies. This
will not only ensure appropriate care at the right cost but also prevent unreasonable demands of suspicious patients and family.

No hospital, business, institution or individual should profiteer from a national calamity such as the COVID­19 pandemic. Hospitals, like any other institution, have a
social responsibility to provide care in times of need. But one should be also aware of the actual costs involved which have to be met. The cost of medical care
often follows the law of diminishing returns; as the treatment gets more sophisticated, further and further increments, although small, cost enormously more. Some of
the drugs used in the care of severely­ill COVID­19 patients may cost more than ₹50,000 a shot, for example, and may provide only a marginally better outcome.

Q­59 According to the passage, why are private hospitals in a larger number than public healthcare?
eng
A. Costing and auditing of care and procedures is not proper in public healthcare.
B. The Indian government spends an abysmally low 1.3% of GDP on public health care.
C. Since Independence, India has, quite rightly, focused attention on the larger picture.

01. Both A and B 02. Only B


03. Both B  and C 04. Only C
05. Only A
Q­59 According to the passage, why are private hospitals in a larger number than public healthcare?
hi
A. Costing and auditing of care and procedures is not proper in public healthcare.
B. The Indian government spends an abysmally low 1.3% of GDP on public health care.
C. Since Independence, India has, quite rightly, focused attention on the larger picture.

01. Both A and B 02. Only B


03. Both B and C 04. Only C
05. Only A
Solution for Question 59 Ans Key: 3
eng Option (3) is the correct answer.
The answer can be inferred from the fourth paragraph.
hi Option (3) is the correct answer.
The answer can be inferred from the fourth paragraph.

Q­60 Choose the word which is most nearly the OPPOSITE in meaning to the word given in bold as used in the passage.
eng
Exorbitant

01. Predominance 02. Extravagant


03. Inexpensive 04. Adhering
05. Impoverish
Q­60 Choose the word which is most nearly the OPPOSITE in meaning to the word given in bold as used in the passage.
hi
Exorbitant
01. Predominance 02. Extravagant
03.  Inexpensive 04. Adhering
05. Impoverish
Solution for Question 60 Ans Key: 3
eng Option (3) is the correct answer.
Exorbitant (adj.)­ (of a price or amount charged) unreasonably high.
For other options:
Predominance (n): command
Impoverish (v): deplete
Adher (v): follow
Extravagant (adj.)­ wasteful
hi Option (3) is the correct answer.
Exorbitant (adj.)­ (of a price or amount charged) unreasonably high.
For other options:
Predominance (n): command
Impoverish (v): deplete
Adher (v): follow
Extravagant (adj.)­ wasteful

Q­61 Choose the word which is most nearly the SAME in meaning to the word given in bold as used in the passage.
eng
Burgeoning

01. Menacing 02. Façade


03. Fortitude 04. Calibrating
05. Proliferating
Q­61 Choose the word which is most nearly the SAME in meaning to the word given in bold as used in the passage.
hi
Burgeoning

01. Menacing 02. Façade


03. Fortitude 04. Calibrating
05. Proliferating
Solution for Question 61 Ans Key: 5
eng Option (5) is the correct answer.
Burgeoning (adj.): increasing
For other words: 
Menacing (adj.): alarming 
Fortitude (n): boldness
Calibrating (v): measure
Proliferating (adj.): increasing
hi Option (5) is the correct answer.
Burgeoning (adj.): increasing
For other words: 
Menacing (adj.): alarming 
Fortitude (n): boldness
Calibrating (v): measure
Proliferating (adj.): increasing

Q­62 What should be suitable title of the passage?
eng

01. Public Healthcare system is a challenge in 02. A transfusion for Ayurveda Centre’s.


India.
03. A transfusion for public healthcare. 04. A transfusion for private healthcare.
05. Better future of Private healthcare.
Q­62 What should be suitable title of the passage?
hi

01. Public Healthcare system is a challenge in 02. A transfusion for Ayurveda Centre’s.


India.
03. A transfusion for  public  healthcare. 04. A transfusion for private healthcare.
05.
05. Better future of Private healthcare.
Solution for Question 62 Ans Key: 3
eng Transfuse means ‘cause (something or someone) to be permeated or infused by something’. As per the passage, public healthcare needs to be infused with
a lot of facilities which have been discussed in the passage. So, option (3) is the most appropriate title of the passage.
hi Transfuse means ‘cause (something or someone) to be permeated or infused by something’. As per the passage, public healthcare needs to be infused with
a lot of facilities which have been discussed in the passage. So, option (3) is the most appropriate title of the passage.

Q­63 Which of the following statements is/are TRUE in the context of the passage?
eng

01. Many a time what is being spent on 02. There are more than one lakh unqualified medical practitioners, mostly in the rural areas.


healthcare gives good results.
03. State­of­the­art corporate hospitals, that 04. Private medicine in India is by no means uniform.
are well equipped and well­staffed and
which provide poor service at high cost.
05. None except (2)
Q­63 Which of the following statements is/are TRUE in the context of the passage?
hi

01. Many a time what is being spent on 02. There are more than one lakh unqualified medical practitioners, mostly in the rural areas.


healthcare gives good results.
03. State­of­the­art corporate hospitals, that 04. Private medicine in India is by no means uniform.
are well equipped and well­staffed and
which provide poor service at high cost.
05. None except  (2)
Solution for Question 63 Ans Key: 5
eng The answer can be inferred from the fifth paragraph.
hi The answer can be inferred from the fifth paragraph.

Q­64 Which statements is/are NOT TRUE according to the passage?
eng
A. Every hospital, business, institution or individual should profiteer from a national calamity such as the COVID­19 pandemic.
B. Hospitals, like any other institution, have a corporate responsibility to provide care in times of need.
C. A huge number of private practitioners and institutions provide a wide range of services of varying quality.

01. Only A 02. Both A and B


03. Only B 04. Both B and C
05. Only C
Q­64 Which statements is/are NOT TRUE according to the passage?
hi
A. Every hospital, business, institution or individual should profiteer from a national calamity such as the COVID­19 pandemic.
B. Hospitals, like any other institution, have a corporate responsibility to provide care in times of need.
C. A huge number of private practitioners and institutions provide a wide range of services of varying quality.

01. Only A 02. Both A  and B


03. Only B 04. Both B and C
05. Only C
Solution for Question 64 Ans Key: 2

eng The answer can be inferred after reading the passage carefully.
hi The answer can be inferred after reading the passage carefully.

Q­65 As per the passage, what can be done to ameliorate public healthcare system in India?
eng

01. Training of doctors and health­care workers 02. Health­care spending by the government must be appropriate, based on evidence, and transparent


also need to be the responsibility of the and accountable.
government mainly.
03. The public health­care system desperately 04. Costing and auditing of care and procedures need to be done by independent bodies.
needs higher government spending.
05. All of the above.
Q­65 As per the passage, what can be done to ameliorate public healthcare system in India?
hi
01. Training of doctors and health­care workers 02. Health­care spending by the government must be appropriate, based on evidence, and transparent
also need to be the responsibility of the and accountable.
government mainly.
03. The public health­care system desperately 04. Costing and auditing of care and procedures need to be done by independent bodies.
needs higher government spending.
05. All of the above
Solution for Question 65 Ans Key: 5

eng Option (5) is the correct answer.
The answer can be inferred after reading the passage carefully.
hi Option (5) is the correct answer.
The answer can be inferred after reading the passage carefully.

Q­66 What is the tone of the passage?
eng

01. Decisive 02. Critical


03. Narrative 04. Cynical
05. Dogmatic
Q­66 What is the tone of the passage?
hi

01. Decisive 02.  Critical


03. Narrative 04. Cynical
05. Dogmatic
Solution for Question 66 Ans Key: 2
eng Critical tone often describes a fault­finding attitude of the author; in a negative sense. It may also indicate a deep analysis of an issue with an impartial
outlook.
hi Critical tone often describes a fault­finding attitude of the author; in a negative sense. It may also indicate a deep analysis of an issue with an impartial
outlook.

Q­67 Why costing and auditing of care and procedures need to be done by independent bodies?
eng

01. Because there is always a fraud or cheating 02. Because government is not conscious in this part.


in government sector.
03. It ensures appropriate care at the right cost 04. Only 1
but also prevent unreasonable demands of
suspicious patients and family.
05. Except 2 and 3
Q­67 Why costing and auditing of care and procedures need to be done by independent bodies?
hi

01. Because there is always a fraud or cheating 02. Because government is not conscious in this part.


in government sector.
03. It ensures appropriate care at the right cost 04. Only 1.
but also prevent unreasonable demands of
suspicious patients and family
05. Except 2 and 3.
Solution for Question 67 Ans Key: 3
eng The answer can be inferred from the end of the sixth paragraph.
hi The answer can be inferred from the end of the sixth paragraph.

Q­68 What is/are the opinion/s of author for ‘Private medicine’?
eng

01. These are developed due to the lack of 02. They have more beds and ventilators as compared to public healthcare.


adequate public healthcare.
03. They were quick to capitalise on good 04. These have successfully engineered a reverse brain drain of many specialists from pursuing lucrative
healthcare demand. jobs abroad and staying back in or returning to India.
05. All are relevant.
Q­68 What is/are the opinion/s of author for ‘Private medicine’?
hi
01. These are developed due to the lack of 02. They have more beds and ventilators as compared to public healthcare
adequate public healthcare
03. They were quick to capitalise on good 04. These have successfully engineered a reverse brain drain of many specialists from pursuing lucrative
healthcare demand jobs abroad and staying back in or returning to India
05. All are relevant.
Solution for Question 68 Ans Key: 5
eng The answer can be inferred from third, fourth and fifth paragraphs.
hi The answer can be inferred from third, fourth and fifth paragraphs.

Q­69 In the following question, three sentences divided into two parts have been given. Choose the parts which have grammatical errors. If all
eng parts of all three sentences are correct, your answer is (5).

I.   The government’s readiness to foot the bill to compensate banks for waiving the interest on interest (A)/ accrued during the moratorium on service loans
of up to Rs 2 crore is welcome relief. (B)
II.   It is unfortunate that the government have not questioned the propriety of the court (P)/ deciding on matters of policy and fiscal allocations (Q)
III.  The suggestion we have made is to inject equity in companies (R)/, with which the companies can service its loans and carry on business (S)

01. A­P­S 02. B­P­S


03. B­Q­R 04. A­Q­S
05. No Error
Q­69 In the following question, three sentences divided into two parts have been given. Choose the parts which have grammatical errors. If all
hi parts of all three sentences are correct, your answer is (5).

I.   The government’s readiness to foot the bill to compensate banks for waiving the interest on interest (A)/ accrued during the moratorium on service loans
of up to Rs 2 crore is welcome relief.(B)
II.   It is unfortunate that the government have not questioned the propriety of the court (P) / deciding on matters of policy and fiscal allocations. (Q)
III.  The suggestion we have made is to inject equity in companies (R)/, with which the companies can service its loans and carry on business.(S)

01. A­P­S 02. B­P­S


03. B­Q­R 04. A­Q­S
05. No Error
Solution for Question 69 Ans Key: 2
eng In option I (B) use ‘servicing’ in place of ‘service’.
In option II (P) use ‘has’ in place of ‘have’ as the subject is singular (government).
In option III (S) use ‘their’ in place of ‘its’ as the subject is plural (companies).
hi In option I (B) use ‘servicing’ in place of ‘service’.
In option II (P) use ‘has’ in place of ‘have’ as the subject is singular (government).
In option III (S) use ‘their’ in place of ‘its’ as the subject is plural (companies).

Q­70 In the following question, three sentences divided into two parts have been given. Choose the parts which have grammatical errors. If all
eng parts of all three sentences are correct, your answer is (5).

I.   Artificial Intelligence will fast become not just a major component of economic competitiveness (A)/ but also a force multiply in strategic capacity.(B) 
II.  Data is oxygen for AI and how data is used to train AI has implications for the data subjects (P)/ which data is utilised for the purpose and for the kind
of algorithm that is produced.(Q)
III.  For India to offer something more than lip service to developed AI (R) /, the first thing to do is to put in place a robust data protection framework. (S)

01. A­P­S 02. B­P­R


03. B­Q­R 04. A­Q­S
05. No Error
Q­70 In the following question, three sentences divided into two parts have been given. Choose the parts which have grammatical errors. If all
hi parts of all three sentences are correct, your answer is (5).

I.   Artificial Intelligence will fast become not just a major component of economic competitiveness (A)/ but also a force multiply in strategic capacity (B) 
II.  Data is oxygen for AI and how data is used to train AI has implications for the data subjects (P)/ which data is utilised for the purpose and for the kind
of algorithm that is produced (Q)
III.  For India to offer something more than lip service to developed AI (R)/, the first thing to do is to put in place a robust data protection framework (S)

01. A­P­S 02. B­P­R


03. B­Q­R 04. A­Q­S
05. No Error
Solution for Question 70 Ans Key: 3
eng In option I (B) use ‘multiplier’ in place of ‘multiply’ a.
In option II (Q) use ‘whose’ in place of ‘which’ a.
In option III (R) use ‘developing’ in place of ‘developed’ a.
hi In option I (B) use ‘multiplier’ in place of ‘multiply’ a.
In option II (Q) use ‘whose’ in place of ‘which’ a.
In option III (R) use ‘developing’ in place of ‘developed’ a.

Q­71 Read the following passage and answer the questions given below:
eng
Education is a very powerful medium to grow in life and perceive something important. In a human’s life, education benefits a lot in decreasing the
difficulties of a hard life. The expertise obtained through the education era encourages everybody regarding their life. Education is a way to enter into
several doors for possibilities to obtain more real prospects in life to improve career growth. Government is also arranging various programs to educate
everyone about education and its benefits in our life especially in rural areas. Education delivers knowledge of equality among everyone in society and
encourages the growth and improvement of the nation.

Which of the following points can be correctly inferred from the given passage?

01. If someone desires to achieve success in 02. The mass belongs to villages or remote areas are not being provided educational benefits.


life, education has the imperceptible role
there.
03. The thought of egalitarianism can be kept 04. If one door is closed, several other doors are also closed, if one is not educated.
alive only through education.
05. Both 1 and 3
Q­71 Read the following passage and answer the questions given below:
hi
Education is a very powerful medium to grow in life and perceive something important. In a human’s life, education benefits a lot in decreasing the
difficulties of a hard life. The expertise obtained through the education era encourages everybody regarding their life. Education is a way to enter into
several doors for possibilities to obtain more real prospects in life to improve career growth. Government is also arranging various programs to educate
everyone about education and its benefits in our life especially in rural areas. Education delivers knowledge of equality among everyone in society and
encourages the growth and improvement of the nation.

Which of the following points can be correctly inferred from the given passage?

01. If someone desires to achieve success in 02. The mass belongs to villages or remote areas are not being provided educational benefits.


life, education has the imperceptible role
there.
03. The thought of egalitarianism can be kept 04. If one door is closed, several other doors are also closed, if one is not educated.
alive only through education
05. Both 1 and 3.
Solution for Question 71 Ans Key: 3
eng From the last line ‘Education delivers knowledge of equality among everyone in society and encourages the growth and improvement of the nation’ of the
passage, the answer is inferred.
hi From the last line ‘Education delivers knowledge of equality among everyone in society and encourages the growth and improvement of the nation’ of the
passage, the answer is inferred.

Q­72 Read the following passage and answer the questions given below:
eng
The Dowry system is evil in society. It has reduced the sacred affair of marriage to a business deal. Brides are treated as a marketable commodity. The
parents of the brides are often put under inhuman pressure for a handsome dowry. Sometimes they become pauper to get their daughters married. In many
cases, young brides are ruthlessly tortured or pushed to suicide. To eradicate this social evil, a total change in the outlook of society is a must. The woman
has to be considered as an equal partner of the man. Widespread education and employment and their rights of property can bring about the right remedy.
Legal prohibition of dowry is not enough. Strong social disapproval of dowry is the only guarantee against this evil. Our youth should come forward to
generate public awareness against this shameful custom.

Which of the following points can be correctly inferred from the given passage?

01. Only through the strictly followed law, 02. When marriages without dowry start taking place in the society, the system of dowry will be


dowry system can be completely eliminated.
exterminated.
03. Until women are not properly educated and 04. The youth of the nation should not overlook the instances happening around and talk about it.
financially independent, the dowry system
will not stopped.
05. Both 2 and 3
Q­72 Read the following passage and answer the questions given below:
hi
The Dowry system is evil in society. It has reduced the sacred affair of marriage to a business deal. Brides are treated as a marketable commodity. The
parents of the brides are often put under inhuman pressure for a handsome dowry. Sometimes they become pauper to get their daughters married. In many
cases, young brides are ruthlessly tortured or pushed to suicide. To eradicate this social evil, a total change in the outlook of society is a must. The woman
has to be considered as an equal partner of the man. Widespread education and employment and their rights of property can bring about the right remedy.
Legal prohibition of dowry is not enough. Strong social disapproval of dowry is the only guarantee against this evil. Our youth should come forward to
generate public awareness against this shameful custom.

Which of the following points can be correctly inferred from the given passage?

01. Only through the strictly followed law, 02. When marriages without dowry start taking place in the society, the system of dowry will be


dowry system can be completely eliminated.
exterminated.
03. Until women are not properly educated and 04. The youth of the nation should not overlook the instances happening around and talk about it.
financially independent, the dowry system
will not stopped.
05. Both 2  and 3
Solution for Question 72 Ans Key: 5
eng Both the points 2 and 3 can be inferred from the given passage.
hi Both the points 2 and 3 can be inferred from the given passage.

Q­73 Read the following passage and answer the questions given below:
eng
Most of the people of our country are farmers, workers, and day laborers who live below the poverty line. But the price of essential commodities is soaring
higher and higher. It has now become impossible for them to make both ends meet. It has severely hit the day laborers, the lower and middle­class families,
and the salaried class too. The prices of rice, vegetables, clothes, mustard oil, medicine, and other essential commodities are also increasing by leaps and
bounds. In most cases, hoarding and black marketing are responsible for such a situation. All these have made the life of the common man miserable. The
government should set up more and more fair price distribution channels for selling essential commodities. The Consumers’ Forum should also be more
active to check such unreasonable hikes.

Which of the following points can be correctly inferred from the given passage?

01. Until the unnecessary gathering and illegal 02. The family who survives on a monthly fixed income faces minimal difficulties when price rise takes


trading is restricted, price rise can’t be place.
stopped.
03. If Price rise is to be controlled, sale of 04. The percentage of wealthy and prosperous people in our nation is more than that of poor or less
mandatorily daily use items through properly affluent people.
scrutinized ways is trader’s responsibility.
05. Both 2 and 4
Q­73 Read the following passage and answer the questions given below:
hi
Most of the people of our country are farmers, workers, and day laborers who live below the poverty line. But the price of essential commodities is soaring
higher and higher. It has now become impossible for them to make both ends meet. It has severely hit the day laborers, the lower and middle­class families,
and the salaried class too. The prices of rice, vegetables, clothes, mustard oil, medicine, and other essential commodities are also increasing by leaps and
bounds. In most cases, hoarding and black marketing are responsible for such a situation. All these have made the life of the common man miserable. The
government should set up more and more fair price distribution channels for selling essential commodities. The Consumers’ Forum should also be more
active to check such unreasonable hikes.

Which of the following points can be correctly inferred from the given passage?

01. Until the unnecessary gathering and illegal 02. The family who survives on a monthly fixed income faces minimal difficulties when price rise takes


trading is restricted, price rise can’t be place.
stopped
03. If Price rise is to be controlled, sale of 04. The percentage of wealthy and prosperous people in our nation is more than that of poor or less
mandatorily daily use items through properly affluent people.
scrutinized ways is trader’s responsibility.
05. Both 2 and 4.
Solution for Question 73 Ans Key: 1

eng Option 1 is very clear from the line ‘In most cases, hoarding and black marketing are responsible for such a situation.’
hi Option 1 is very clear from the line ‘In most cases, hoarding and black marketing are responsible for such a situation.’

Q­74 Read the following passage and answer the questions given below:
eng
We all know that health is wealth. With its intricate network of bones, muscles, and organs, a well­functioning human body is much like an orchestrated
symphony. To keep this orchestra playing well, we need physical exercise. It may take the form of sports, yoga, or even regular walking. It is well­known
that people who engage in physical exercise stay happier and live longer. Our society is turning towards more and more technical sophistication and
automation. The machine has replaced our physical labor. To compensate for this change in lifestyle we need physical exercise. Exercise also sharpens our
intellect. It keeps a balance between our body and mind. With the help of regular physical training, we will stay healthier, happier, and more alert. However,
over­exercise or exercising in an improper way may tell upon our health and growth. We must therefore do it in a balanced form.

Which of the following points can be correctly inferred from the given passage?

01. If one wants to survive for a long time with 02. The more physically one gets active, the more one becomes wise and sagacious.


no or less stress, one must sweat out.
03. The surge in the evolution and use of 04. Any action if not performed with carelessness may result into negative outcomes.
machine does not strike at the importance
human workers.
05. Both 1 and 2
Q­74 Read the following passage and answer the questions given below:
hi
We all know that health is wealth. With its intricate network of bones, muscles, and organs, a well­functioning human body is much like an orchestrated
symphony. To keep this orchestra playing well, we need physical exercise. It may take the form of sports, yoga, or even regular walking. It is well­known
that people who engage in physical exercise stay happier and live longer. Our society is turning towards more and more technical sophistication and
automation. The machine has replaced our physical labor. To compensate for this change in lifestyle we need physical exercise. Exercise also sharpens our
intellect. It keeps a balance between our body and mind. With the help of regular physical training, we will stay healthier, happier, and more alert. However,
over­exercise or exercising in an improper way may tell upon our health and growth. We must therefore do it in a balanced form.

Which of the following points can be correctly inferred from the given passage?

01. If one wants to survive for a long time with 02. The more physically one gets active, the more one becomes wise and sagacious.


no or less stress, one must sweat out.
03. The surge in the evolution and use of 04. Any action if not performed with carelessness may result into negative outcomes.
machine does not strike at the importance
human workers.
05. Both 1 and  2
Solution for Question 74 Ans Key: 5
eng Both the points 1 and 2 can be easily inferred from the given passage.
hi Both the points 1 and 2 can be easily inferred from the given passage.

Q­75 Read the following passage and answer the questions given below:
eng
In the recent period, many campaigns have been organized against employing children in mines, factories, motor garages, restaurants, tea stalls, and shops.
There are various households that employ poor children though only a few provide their child­servants with adequate food and clothing. They are deprived
of education and their wages are paid to their parents and the money is rarely spent on them. It is the extreme poverty of their parents that forces the
children to adopt such a profession at a minor age. Hence, we need, not only strict laws but also a thorough economic reform to reduce the poverty line. At
the same time, the spread of literacy is essential to root out these shameful practices still prevalent in our country.

Which of the following points can be correctly inferred from the given passage?

01. This is the fact that at some extent, the weak 02. In most of the houses where children are kept as servant, they are offered delicious items to eat and


financial condition of a family brings a child good dresses to wear.
on the miserable path of earning.
03. Poor working children are able to continue 04. If any nation wishes to eradicate poverty from there, it needs to do nothing else but making strict
their studies with the help of the money paid laws.
to them against their services.
05. Both 2 and 3
Q­75 Read the following passage and answer the questions given below:
hi
In the recent period, many campaigns have been organized against employing children in mines, factories, motor garages, restaurants, tea stalls, and shops.
There are various households that employ poor children though only a few provide their child­servants with adequate food and clothing. They are deprived
of education and their wages are paid to their parents and the money is rarely spent on them. It is the extreme poverty of their parents that forces the
children to adopt such a profession at a minor age. Hence, we need, not only strict laws but also a thorough economic reform to reduce the poverty line. At
the same time, the spread of literacy is essential to root out these shameful practices still prevalent in our country.

Which of the following points can be correctly inferred from the given passage?

01. This is the fact that at some extent, the weak 02. In most of the houses where children are kept as servant, they are offered delicious items to eat and


financial condition of a family brings a child good dresses to wear
on the miserable path of earning.
03. Poor working children are able to continue 04. If any nation wishes to eradicate poverty from there, it needs to do nothing else but making strict
their studies with the help of the money paid laws
to them against their services
05. Both 2 and 3
Solution for Question 75 Ans Key: 1
eng From the line ‘It is the extreme poverty of their parents that forces the children to adopt such a profession at a minor age’ the answer can be inferred.
hi From the line ‘It is the extreme poverty of their parents that forces the children to adopt such a profession at a minor age’ the answer can be inferred.

Q­76 In the following question a table divided into two columns is given. In these columns the parts of three sentences (first part in Column I) and
eng (second part in Column II) are given but not in order, you need to match the parts to make the meaningful sentences.

01. P­A, Q­C, R­B 02. P­B, Q­A, R­C


03. P­C, Q­A, R­B 04. P­A, Q­B, R­C
05. P­C, Q­B, R­A
Q­76 In the following question a table divided into two columns is given. In these columns the parts of three sentences (first part in Column I) and
hi (second part in Column II) are given but not in order, you need to match the parts to make the meaningful sentences.

01. P­A, Q­C, R­B 02. P­B, Q­A, R­C


03. P­C, Q­A,  R­B 04. P­A, Q­B, R­C
05. P­C, Q­B, R­A
Solution for Question 76 Ans Key: 3

eng The correct sentences are:
Since Russia is the world’s third­largest oil producer after the US and Saudi Arabia, this resulted in a price rise of crude oil because of less supply and the
same demand.
A rise in crude oil prices means petrol and diesel prices may increase, which can result in price rise of everyday commodities such as grains and vegetables
because of the rise in the cost of transporting these commodities.
Global semiconductor supply is already under stress due to increasing demand for consumer products that contain chips and also because of disruption in
production during pandemics.
hi The correct sentences are:
Since Russia is the world’s third­largest oil producer after the US and Saudi Arabia, this resulted in a price rise of crude oil because of less supply and the
same demand.
A rise in crude oil prices means petrol and diesel prices may increase, which can result in price rise of everyday commodities such as grains and vegetables
because of the rise in the cost of transporting these commodities.
Global semiconductor supply is already under stress due to increasing demand for consumer products that contain chips and also because of disruption in
production during pandemics.

Q­77 In the following question a table divided into two columns is given. In these columns the parts of three sentences (first part in Column I) and
eng (second part in Column II) are given but not in order, you need to match the parts to make the meaningful sentences.
01. P­A, Q­C, R­B 02. P­B, Q­A, R­C
03. P­C, Q­A, R­B 04. P­A, Q­B, R­C
05. P­C, Q­B, R­A
Q­77 In the following question a table divided into two columns is given. In these columns the parts of three sentences (first part in Column I) and
hi (second part in Column II) are given but not in order, you need to match the parts to make the meaningful sentences.

01. P­A,  Q­C, R­B 02. P­B, Q­A, R­C


03. P­C, Q­A, R­B 04. P­A, Q­B, R­C
05. P­C, Q­B, R­A
Solution for Question 77 Ans Key: 1
eng The correct sentences are:
As the internet offers more and more opportunities for income, the digital divide can increase income inequality
between digital literates and digital illiterates.
As the extraction of data and its misuse is increasing rapidly, everyone is of the opinion that there should be more
privacy for our data on the internet.
At present, many companies are extracting data from us through our online presence and are using it to give us a
more personalized experience.
hi The correct sentences are:
As the internet offers more and more opportunities for income, the digital divide can increase income inequality
between digital literates and digital illiterates.
As the extraction of data and its misuse is increasing rapidly, everyone is of the opinion that there should be more
privacy for our data on the internet.
At present, many companies are extracting data from us through our online presence and are using it to give us a
more personalized experience.

Q­78 In the following question a table divided into two columns is given. In these columns the parts of three sentences (first part in Column I) and
eng (second part in Column II) are given but not in order, you need to match the parts to make the meaningful sentences.

01. P­A, Q­C, R­B 02. P­B, Q­A, R­C


03. P­C, Q­A, R­B 04. P­B, Q­C, R­A
05. P­C, Q­B, R­A
Q­78 In the following question a table divided into two columns is given. In these columns the parts of three sentences (first part in Column I) and
hi (second part in Column II) are given but not in order, you need to match the parts to make the meaningful sentences.
01. P­A, Q­C, R­B 02. P­B, Q­A, R­C
03. P­C, Q­A, R­B 04. P­B , Q­C, R­A
05. P­C, Q­B, R­A
Solution for Question 78 Ans Key: 4
eng The correct sentences are:

Some factors such as the size of the international transactions in that currency, financial stability and political stability of the country etc., determine whether
the currency can become an international currency or not.
The political stability of India can be very helpful in gaining the confidence of foreign investors and thereby can push for the internationalization of the Rupee.
Making trade agreements with other countries in Rupees can increase the share of the Rupee in international transactions and hence can pave a path to the
internationalization of the Rupee.
hi The correct sentences are:

Some factors such as the size of the international transactions in that currency, financial stability and political stability of the country etc., determine whether
the currency can become an international currency or not.
The political stability of India can be very helpful in gaining the confidence of foreign investors and thereby can push for the internationalization of the Rupee.
Making trade agreements with other countries in Rupees can increase the share of the Rupee in international transactions and hence can pave a path to the
internationalization of the Rupee.

Q­79 In the following question a table divided into two columns is given. In these columns the parts of three sentences (first part in Column I) and
eng (second part in Column II) are given but not in order, you need to match the parts to make the meaningful sentences.

01. P­A, Q­C, R­B 02. P­B, Q­A, R­C


03. P­C, Q­A, R­B 04. P­A, Q­B, R­C
05. P­C, Q­B, R­A
Q­79 In the following question a table divided into two columns is given. In these columns the parts of three sentences (first part in Column I) and
hi (second part in Column II) are given but not in order, you need to match the parts to make the meaningful sentences.

01. P­A, Q­C, R­B 02. P­B, Q­A, R­C


03.  P­C, Q­A,R­B 04. P­A, Q­B, R­C
05. P­C, Q­B, R­A
Solution for Question 79 Ans Key: 3

eng The correct sentences are:
Due to high sugars or salts or fats in fast foods, the regular consumers will have to face several health issues such as high blood pressure, obesity and other
lifestyle diseases.
The pre­prepared foods save a lot of time. In hectic lifestyles, saving time is a priority and hence the consumption of fast foods is increasing.
Many chains are now using claims about nutrient content and health benefits on their websites to create a marketing edge and perhaps make us feel less
guilty about our next fast food purchase.
hi The correct sentences are:
Due to high sugars or salts or fats in fast foods, the regular consumers will have to face several health issues such as high blood pressure, obesity and other
lifestyle diseases.
The pre­prepared foods save a lot of time. In hectic lifestyles, saving time is a priority and hence the consumption of fast foods is increasing.
Many chains are now using claims about nutrient content and health benefits on their websites to create a marketing edge and perhaps make us feel less
guilty about our next fast food purchase.

Q­80 In the following question a table divided into two columns is given. In these columns the parts of three sentences (first part in Column I) and
eng (second part in Column II) are given but not in order, you need to match the parts to make the meaningful sentences.

01. P­A, Q­C, R­B 02. P­B, Q­A, R­C


03. P­C, Q­A, R­B 04. P­A, Q­B, R­C
05. P­C, Q­B, R­A
Q­80 In the following question a table divided into two columns is given. In these columns the parts of three sentences (first part in Column I) and
hi (second part in Column II) are given but not in order, you need to match the parts to make the meaningful sentences.

01. P­A, Q­C, R­B 02. P­B, Q­A, R­C


03. P­C, Q­A, R­B 04. P­A, Q­B, R­C
05. P­C, Q­B,  R­A
Solution for Question 80 Ans Key: 5
eng The correct sentences are:
Greenwashing is a marketing practice that misleads consumers by making them believe that their products are environment­friendly when in reality they are
not.
Many companies claim that their products are eco­friendly and they are helping the planet to mislead consumers into choosing their products.
More and more companies are unnecessarily using nature images, green colours in display and video advertisements to trick consumers into believing that
their products are environmentally friendly.
hi The correct sentences are:
Greenwashing is a marketing practice that misleads consumers by making them believe that their products are environment­friendly when in reality they are
not.
Many companies claim that their products are eco­friendly and they are helping the planet to mislead consumers into choosing their products.
More and more companies are unnecessarily using nature images, green colours in display and video advertisements to trick consumers into believing that
their products are environmentally friendly.
Passage for Question 81, 82, 83, 84, 85 (ST­18 CWEPOMAINS 2017(21­24))
eng Passage

Read the following information carefully to answer the questions asked­

Four countries ­ USA, Kenya, Russia and Australia ­ participated in a 4 × 400 metres relay which is an athletic track event in which each team comprises four
runners, each of whom completes 1 lap of 400 metres. In the event, four runners, with one from each team, start running simultaneously from the same point on the
track. As soon as the first runner from any team completes 1 lap of 400 metres, the second runner from that team starts running, from the same starting point, and
this process continues till the fourth runner from the team has completed his lap. The same goes for the other teams .The winner of the event is the team whose
players take the least composite times to complete the four laps. The following bar graph gives the information about the average speeds at which runners from the
four teams ran to complete their laps.

hi Passage

पूछे गए का उ र देने के िलए दी गयी जानकारी का यानपूवक अ ययन क िजये  ­

चार देश  ­ संयु रा य अमे रका, के या,  स और ऑ ेिलया न एक 4 × 400 मीटर रले म भाग िलया जो एक एथले टक ैक इवट है िजसम येक टीम म चार धावक होते ह,
िजनम से येक च  400 मीटर दूरी का होता है इस ितयोिगता म येक टीम से एक धावक एक ही बंदु से एक साथ ैक पर दौड़ना ार भ करते ह । जैसे ही कसी भी टीम
का थम धावक 400 मीटर क दूरी का थम च पूरा करता है, वही शु आती बंदु से उस टीम का दूसरा धावक दौड़ना ार भ करते है, और यह या तब तक जारी रहती है
जब टीम का चौथा धावक अपना च पूरा कर लेता है। यह अ य टीम के िलए भी समान होता है। इस ितयोिगता क िवजेता वह टीम है िजसके िखलाड़ी सम प से कम
समय लेते ह । िन िलिखत रेखा आरेख म धावक क औसत गित के बारे म जानकारी दी गई है, िजसम चार टीम के धावक अपना च पूरा करने के िलए लेते ह ।

Q­81 Which country won the event?
eng

01.  Australia 02. Russia


03.  USA 04. Kenya
05. Can not be determined
Q­81 कस देश ने ितयोिगता जीती ?
hi

01. ऑ ेिलया 02. स


03. संयु रा य अमे रका 04. के या
05. िनधा रत नही कया जा सकता
Solution for Question 81 Ans Key: 3
eng

hi

Q­82 Which of the following statements is definitely true?
eng

01. A runner from Australia was never behind 02. Third runner from Russia and Kenya never met during the course of the event.


the corresponding numbered runner from
USA
03. Second runner from Kenya and Russia 04. A runner from Kenya was always behind the corresponding numbered runner from Russia.
definitely met once during the event.
05. All are definitely false
Q­82 िन म से कौन सा कथन िनि त प से सही है? 
hi

01. ऑ ेिलया से एक धावक संयु रा य अमे रका 02. स और के या से तीसरा धावक इस घटना के दौरान कभी नह िमले।
से इसी नंबर वाले धावक के पीछे कभी नह था
03. घटना के दौरान के या और स से दूसरे धावक 04. के या का धावक हमेशा स से समान म के धावक के पीछे था
एक बार िमले थे
05. सभी िनि त प से गलत ह
Solution for Question 82 Ans Key: 4
eng all are false except

A runner from Kenya was always behind the corresponding numbered runner from Russia.
hi सभी गलत ह िसवाय

के या का धावक हमेशा स से समान म के धावक के पीछे था

Q­83 Which country was second  country to finish second lap?
eng

01. Russia 02. Kenya


03. Austraila 04. USA
05. Can not be determined
Q­83 दूसरा च पूरा करने वाला दूसरा देश कौन सा था?
hi

01. स 02. के या
03. ऑ ेिलया 04. संयु रा य अमे रका
05. िनधा रत नह कया जा सकता है
Solution for Question 83 Ans Key: 3
eng USA    80.8

Kenya   83.33

Russia   66.67

Austraila   70.76
hi सं य ु  रा य अमे रका  80.8

के या   83.33
स   66.67

ऑ े ि लया  70.76

Q­84 When second runner from Kenya started running second runner from Russia was how meter ahead?
eng

01. 66.67 meter 02. 40 meter


03. 33.33 meter 04. 83.33 meter
05. Can not be determined
Q­84 जब के या का दूसरा धावक दौड़ शु कर रहा था, तो स का दूसरा धावक कतने मीटर आगे था?
hi

01. 66.67 मीटर 02. 40 मीटर


03. 33.33 मीटर 04. 83.33 मीटर
05. िनधा रत नह कया जा सकता है
Solution for Question 84 Ans Key: 1
eng Russia;s first runner took 400/10=40 sec

when Russia;s runner finished kenya's runner was 400­40×9=40 meter behind

so the time Russia's second runner has  40/9

required answer=

hi स के पहले धावक ारा िलया गया समय  400/10=40 sec

जब स के थम धावक ने दौड़ समा क के या का धावक 400­40×9=40 मीटर पीछे था

अतः स के दूसरे धावक क पास समय है 40/9

अिभ ट उ र=

Q­85 What was the shortest distance between the starting point and the point at which the second runner from USA and Kenya met?
eng

01. 100 metres 02. 50 metres


03. 66.67 metres 04. 244.44 metres
05. Second runner from USA and Kenya did
not meet
Q­85 िजस समय संयु रा य अमे रका और के या के ि तीय धावक िमले थे उस बंदु के और ारंिभक बंदु बीच यूनतम दूरी या थी?
hi

01. 100 मीटर 02. 50 मीटर


03. 66.67 मीटर 04. 244.44 मीटर
05. संयु रा य अमे रका और के या का दूसरा
धावक कभी नह िमले
Solution for Question 85 Ans Key: 5
eng Second runner from USA and Kenya did not meet

hi संयु  रा य अमे रका और के या का दूसरा धावक कभी नह ं िमले

Passage for Question 86, 87, 88, 89, 90 (ST­2037 SBIPO MAINS M (11­15) 2018)
eng Passage

Study the following graph carefully to answer the given questions
Total downstream distance=200 Kms (Distance provided in charts are in percentage)
Total upstream distance=300 Kms

hi Passage

दए गए के उ र दे न े के ि ल ए िन िलिखत ाफ क ा यानपू व क अ ययन कर ­


कुल अनु वाह दूरी=200  कमी (चाट मे दी गई दूरी ितशत मे है)

कुल उ व वाह दूरी=300  कमी


Q­86 Total downstream distance travelled by boat on Monday, Tuesday and Wednesday together is approximately what percentage of total upstream distance
eng travelled by boat on the same days together?

01. 65% 02. 38%


03. 49% 04. 32%
05. 55%
Q­86 सोमवार, मंगलवार और बुधवार को नाव ारा या ा क गई कुल अनु वाह दूरी उ ह दन म नाव ारा या ा क गई कुल ऊ व वाह दूरी का लगभग कतना ितशत
hi है?

01. 65% 02. 38%


03. 49% 04. 32%
05. 55%
Solution for Question 86 Ans Key: 3
eng Total downstream distance travelled by boat on Monday, Tuesday and Wednesday = 108
Total up stream distance travelled by boat on Monday, Tuesday and Wednesday = 222
Required percent = 

hi सोमवार, मंगलवार और बुधवार को नाव ारा या ा क गई कुल अनु वाह दूरी = 108 


सोमवार, मंगलवार और बुधवार को नाव ारा या ा क गई कुल ऊ व वाह दूरी = 222 
अभी ितशत = 

Q­87 A boatman rows downstream at 6 kmph on Wednesday. Find the time taken by the boat to cover upstream distance on the same day.
eng

01. 36 hours 02. 24 hours


03. 30 hours 04. 48 hours
05. None of these
Q­87 एक नािवक बुधवार को अनु वाह 6  कमी ित घंटे क चाल से नाव चलाता है। उसी दन ऊ व वाह दूरी को तय करने के िलए नाव ारा िलए गए समय को ात
hi क िजये।

01. 36 घंटे 02. 24 घंटे


03. 30 घंटे 04. 48 घंटे
05. इनम से कोई नह
Solution for Question 87 Ans Key: 1
eng Downstream Speed = 6kmph=(4+2)
Upstream Speed = (4­2) kmph = 2kmph
Time taken by the boat to cover upstream distance = 72/2 = 36 hours
hi अनु वाह चाल = 6 िकमी ित घंटा=(4+2)
ऊ व वाह चाल = (4-2) िकमी ित घंटा = 2 िकमी
ऊ व वाह दरू ी को तय करने के िलए नाव ारा िलया गया समय = 72/2 = 36 घंटे

Q­88 Find the difference between the upstream distance travelled by boat on Tuesday and Friday together and the downstream distance travelled by boat on the
eng same days together.

01. 12 km 02. 18 km


03. 16 km 04. 14 km
05. None of these
Q­88 मंगलवार और शु वार को नाव ारा या ा क गई ऊ व वाह दूरी और उ ह दन म एकसाथ िमलाकर नाव ारा या ा क गई अनु वाह दूरी के बीच अंतर ात
hi क िजये।

01. 12  कमी. 02. 18  कमी.


03. 16  कमी. 04. 14  कमी.
05. इनम से कोई नह
Solution for Question 88 Ans Key: 4
eng Required Difference = 96 – 82 = 14
eng Required Difference = 96 – 82 = 14
hi अभी दरू ी = 96 – 82 = 14

Q­89 Find the ratio between the down stream distance travelled by boat on Monday, Wednesday, Thursday and Friday together and the Up stream distance
eng travelled by the boat on the same days together?

01. 15 : 27 02. 14 : 21


03. 20 : 27 04. 17 : 24
05. None of these
Q­89 सोमवार, बुधवार, गु वार और शु वार को नाव ारा या ा क गई अनु वाह दूरी का उ ह दन म एकसाथ नाव ारा या ा क गई ऊ व वाह दूरी के बीच अनुपात
hi ात क िजये?

01. 15 : 27 02. 14 : 21


03. 20 : 27 04. 17 : 24
05. इनम से कोई नह
Solution for Question 89 Ans Key: 4
eng Required ratio =170:240=17 : 24.
hi अभी अनुपात = 170 : 240 = 17 : 24.

Q­90 A boatman rows up stream at 16 kmph on Tuesday. Find the approximate time taken by the boat to cover downstream distance and upstream distance on
eng the same day.

01. 3.5 hours 02. 4.25 hours


03. 4.75 hours 04. 5.25 hours
05. 3.75 hours
Q­90 एक नािवक मंगलवार को 16  कमी ित घंटे क चाल से ऊ व वाह नाव चलाता है । उसी दन अनु वाह दूरी और ऊ व वाह दूरी को तय करने के िलए नाव ारा िलया
hi गया अनुमािनत समय ात क िजये।

01. 3.5 घंटे 02. 4.25 घंटे


03. 4.75 घंटे 04. 5.25 घंटे
05. 3.75 घंटे
Solution for Question 90 Ans Key: 3
eng U=u­v=16
v=6
u=22
D=28
Required answer =   = hours

hi U=u­v=16
v=6
u=22
D=28
अभी उ र =   = घंटे

Q­91 Each of these questions consists of a question followed by information in three statements. You have to study the question and the statements and decide
eng that information in which of the statement(s) is/are necessary to answer the question.

Find in how many days Akash will complete the work­

Statement I: Ankita and Akash working together completes the whole work in 52days.
Statement II: Ankita working with Priyansh completes 75% work in 45 days.
Statement III: Efficiency of Priyansh is 20% less than efficiency of Ankita.

01. Any two 02. I and II or III


03. III and I or II 04. All three together are not sufficient
05. All three together are sufficient
Q­91 नीचे येक म, एक और उसके बाद तीन कथन म जानकारी दी गई है। आप ो और कथन का अ ययन क िजए और यह तय क िजये क कौनसा/से कथन म
hi दी गई जानकारी का उ र देने के िलए आव यक है/ह।
आकाश कतने दन म काय समा करेगा ­

कथन I: अं कता और आकाश एक साथ काय करते ए 52  दन म पूरे काय को समा करते है।
कथन II: अं कता ि यांश के साथ काय करते ए 45  दनो म 75% काय पूरा करती है।
कथन III: ि यांश क मता अं कता से 20% कम है।

01. कोई भी दो 02. I तथा II या III


03. III तथा I या II 04. तीनो एक साथ पया नह ह
05. तीनो एक साथ पया ह
Solution for Question 91 Ans Key: 5
eng All three together are sufficient
hi तीनो एक साथ पया ह

Q­92 Each of these questions consists of a question followed by information in three statements. You have to study the question and the statements and decide
eng that information in which of the statement(s) is/are necessary to answer the question.

Find the average age of a 4 member family­

Statement I­Ragini is sister of Shyam and difference of their age is 4 years.
Statement II –Ragini’s father is 8 years elder to her mother.
Statement III­ Ratio of age of Shyam’s father and mother is 6:5.

01. Any two 02. I and II or III


03. II and I or III 04. All three together are sufficient.
05. All three together are not sufficient
Q­92 नीचे येक म, एक और उसके बाद तीन कथन म जानकारी दी गई है। आप ो और कथन का अ ययन क िजए और यह तय क िजये क कौनसा/से कथन
hi म दी गई जानकारी का उ र देने के िलए आव यक है/ह।

4 सद यीय प रवार क औसत आयु ात क िजये  ­

कथन I­­रािगनी याम क बहन है और उनक उ का अ तर 4 वष है।


कथन II –रािगनी के िपता उसक मां से 8 वष बड़े ह।
कथन III­  याम के िपता और मां क आयु का अनुपात 6: 5 है।

01. कोई भी दो 02. I तथा II या III


03. II तथा I या III 04. तीनो एक साथ पया ह
05. तीनो एक साथ पया नह ह
Solution for Question 92 Ans Key: 5
eng All three together are not sufficient.
hi तीनो एक साथ पया नह ह

Q­93 Each of these questions consists of a question followed by information in three statements. You have to study the question and the statements and decide
eng that information in which of the statement(s) is/are necessary to answer the question.

Find the average of six consecutive even numbers­

Statement I­Difference of first and fifth number is 8.
Statement II­ Difference of third and sixth number is 6.
Statement III­Fifth number is 6.25% of 800.

01. Any two 02. I and II or III


03. Only III 04. III and I or III
05. None of these
Q­93 नीचे येक म, एक और उसके बाद तीन कथन म जानकारी दी गई है। आप ो और कथन का अ ययन क िजए और यह तय क िजये क कौनसा/से कथन
hi म दी गई जानकारी का उ र देने के िलए आव यक है/ह ।

छह मागत सम सं या का औसत ात क िजये  ­

कथन I­पहली और पांचव सं या का अंतर 8 है ।


कथन II­ तीसरी और छठी सं या का अंतर 8 है ।
कथन III­पांचवी सं या 800 क  6.25%  है ।

01. 02.
01. कोई भी दो 02. I तथा II या III
03. केवल III 04. III तथा I या III
05. इनमे से कोई नह
Solution for Question 93 Ans Key: 3
eng Only III is sufficient

hi केवल III पया है

Q­94 Each of these questions consists of a question followed by information in three statements. You have to study the question and the statements and decide
eng that information in which of the statement(s) is/are necessary to answer the question.

Find the profit percentage of shopkeeper­

Statement I­Shopkeeper marked the price of an article at 80% more than its CP.
Statement II­MP of article is 20% more than its SP.
Statement III­Shopkeeper gives total discount of 960 Rs to his customer.

01. I and II 02. I and III


03. II and I or III 04. Any two
05. None of these
Q­94 नीचे येक म, एक और उसके बाद तीन कथन म जानकारी दी गई है। आप ो और कथन का अ ययन क िजए और यह तय क िजये क कौनसा/से कथन
hi म दी गई जानकारी का उ र देने के िलए आव यक है/ह ।

दुकानदार का लाभ ितशत ात क िजये  ­

कथन I­दुकानदार व तु का अं कत मू य यमु य से 80% बढ़ाकर िलखता है ।


कथन II­व तु का अं कत मू य िव य मू य से  20% अिधक है ।
कथन III­दुकानदार अपने ाहक को कुल 960  पये क छूट देता है ।

01. I तथा II 02. I तथा III


03. II तथा I या III 04. कोई भी दो
05. इनमे से कोई नह
Solution for Question 94 Ans Key: 1
eng I and II are sufficient.

hi I तथा II पया ह

Q­95 Each of these questions consists of a question followed by information in three statements. You have to study the question and the statements and decide
eng that information in which of the statement(s) is/are necessary to answer the question.

Find the compound interest of 2 years and 3 months compounded annually­

Statement I:­If the principal is invested for 3 years at SI it generates a interest equal to 1200 rs.
Statement II­ CI of second years is 480 Rs.
Statement III­ CI of third year is 96 Rs more than CI of second year.

01. Any two 02. I and (II or III)


03. II and III 04. I and III
05. None of these
Q­95 नीचे येक म, एक और उसके बाद तीन कथन म जानकारी दी गई है। आप ो और कथन का अ ययन क िजए और यह तय क िजये क कौनसा/से कथन म
hi दी गई जानकारी का उ र देने के िलए आव यक है/ह।

2 वष तथा 3 महीने का च वृि याज ात क िजये य द याज वा षक िनयोिजत होता है  ­

कथन I:­य द मूलधन को साधारण याज पर 3 वष के िलए िनवेश कया जाता है तो यह 1200  पये के बराबर याज उ प करता है ।
कथन II­ दूसरे वष का च वृि याज 480  पये है ।
कथन III­ तीसरे वष का च वृि याज दूसरे वष से 96  पये अिधक है ।

01. कोई भी दो 02. I तथा (II या III)


03. II तथा III 04. I तथा III
05. इनमे से कोई नह
Solution for Question 95 Ans Key: 2
eng I and (II or III) are sufficient

hi I तथा (II या III) पया ह

Q­96 Find the correct relationship between two quantities.
eng
Quantity I­ A box contains 5 green balls, 3 yellow balls and 3 white balls. What is the probability that they are of different color if 3 balls picked at
random?
Quantity II­  A card is drawn at random from a deck. Find the probability to getting either an king or an ace?

01. Quantity I > Quantity II 02. Quantity I ≥  Quantity II


03. Quantity II > Quantity I 04. Quantity II ≥ Quantity I
05. Quantity I = Quantity II or Relation cannot
be established
Q­96 द ो मा ा के बीच सही सं ब ं ध थािपत क िजये ।
hi
मा ा I­एक बॉ स म 5 हरी गद,3 पीली गद और 3 सफेद गद ह।य द 3 गद को यादृि छक प से चुना जाए तो उनके अलग­अलग रंग होने क ाियकता या ह?
मा ा II­ एक ताश क ग ी से एक प े को अिनयिमत प से िनकला जाता है। एक बादशाह या एक इ ा होने क ाियकता ात क िजये?

01. मा ा I > मा ा II 02. मा ा I ≥ मा ा II


03. मा ा II > मा ा I 04. मा ा II ≥ मा ा I
05. मा ा I = मा ा II या संबध
ं थािपत नह कया
जा सकता है
Solution for Question 96 Ans Key: 3

eng
Quantity I­ required probability = 

Quantity II­ required probability = 

hi
मा ा I-आव यक ाियकता = 

मा II- आव यक ाियकता = 

Q­97 Find the correct relationship between two quantities.
eng
Quantity I­ When a motorboat travels in upstream then its speed become 37.5% of the speed of the motorboat in still water .The speed of the stream is
how much percentage of the speed of the motorboat in still water?
Quantity II­ When a sum of money was invested at simple interest then at the end of 20 years the amount becomes 1350% of the sum of money. What is
the rate of interest?

01. Quantity I > Quantity II 02. Quantity I ≥  Quantity II


03. Quantity I >  Quantity II 04. Quantity II ≥ Quantity I
05. Quantity I = Quantity II or Relation cannot
be established
Q­97 द ो मा ा के बीच सही सं ब ं ध थािपत क िजये ।
hi
मा ा I­ जब एक मोटरबोट धारा के िवपरीत दशा म या ा करती है तो उसक गित ि थर जल म मोटरबोट क गित का 37.5% हो जाती है। धारा क गित,ि थर जल
म मोटरबोट क गित का कतना ितशत है?
मा ा II­ जब साधारण याज पर धनरािश का िनवेश कया जाता है तो 20 वष के अंत म यह मूलधन का 1350% हो जाती है। याज क दर या है?

01. मा ा I > मा ा II 02. मा ा I ≥ मा ा II


03. मा ा II > मा ा I 04. मा ा II ≥ मा ा I
05. मा ा I = मा ा II या संबध
ं थािपत नह कया
जा सकता है
Solution for Question 97 Ans Key: 5
eng Quantity I­ Let the speed of the motorboat in still water = x km per hour and the speed of the stream = y km per hour.
according to the question, x – y =37.5%of x =>   then required percentage=

Quantity II­ let the some of money be x and rate be r%  then x×20×   = 13.5x => r= 62.5%

hi मा ा I – माना ि थर जल म मोटरबोट क गित = x िकमी ित घंटा और धारा क गित = y िकमी ित घंटा है।

के अनुसार, x - y = X का 37.5%  =>  तब आव यक ितशत =

मा ा II- माना रािश x है और याज क दर r% है, तब x×20×   = 13.5x => r= 62.5%

Q­98 Find the correct relationship between two quantities.
eng
Quantity I­ From a group of 6 consonant and 4 vowels how many words can be formed by taking 3consonent and 2vowels?
Quantity II­ In how many way the word “BENEVOLENT” can be arrange if all vowels comes together?

01. Quantity I > Quantity II 02. Quantity I ≥  Quantity II


03. Quantity II > Quantity I 04. Quantity II ≥ Quantity I
05. Quantity I = Quantity II or Relation cannot
be established
Q­98 द ो मा ा के बीच सही सं ब ं ध थािपत क िजये ।
hi
मा ा I­ 6  ज
ं न और 4  वर के समूह से 3  ज
ं न और 2  वर लेकर कतने श द बनाये जा सकते है?
मा ा II­ श द “BENVOLENT” के अ र का योग कर ऐसे कतने श द बनाया जा सकता है जब क सभी वर एक साथ आये?

01. मा ा I > मा ा II 02. मा ा I ≥ मा ा II


03. मा ा II > मा ा I 04. मा ा II ≥ मा ा I
05. मा ा I = मा ा II या संबध
ं थािपत नह कया
जा सकता है
Solution for Question 98 Ans Key: 1

eng Quantity I­

Quantity II­

hi मा ा I­
मा ा II­

Q­99 Find the correct relationship between two quantities.
eng
Quantity I­  In how many days A can alone do the work if A and B can do a work in 60 days, B and C can do a work in 45 days, C and A can do work
in 30 days.
Quantity II­ A and B together can complete three fourth of a work in 45days. They starts work together but after 20days A left the work and remaining
work completed by B in 100 days. In how many days A can alone complete the total work ?

01. Quantity I > Quantity II 02. Quantity I ≥  Quantity II


03. Quantity II > Quantity I 04. Quantity II ≥ Quantity I
05. Quantity I = Quantity II or Relation cannot
be established
Q­99 द ो मा ा के बीच सही सं ब ं ध थािपत क िजये ।
hi
मा ा I­ A  कतने दन म अकेले काय कर सकता है य द A और Bिमलकर कसी काय को 60  दन म कर सकते ह,Bऔर C उसी काय को 45  दन म,C और A उसी
काय को 30  दन म काय कर सकते ह।
मा ा II­A और B एक साथ 45  दन म कसी काम का तीन चौथाई भाग पूरा कर सकते ह। वे एक साथ काम करना ारंभ करते ह ले कन 20  दन के बाद A ने काम
छोड़ दया और शेष काय B  ारा 100  दन म पूरा कया गया। A कुल काय को कतने दन म पूरा कर सकता है?

01. मा ा I > मा ा II 02. मा ा I ≥ मा ा II


03. मा ा II > मा ा I 04. मा ा II ≥ मा ा I
05. मा ा I = मा ा II या संबध
ं थािपत नह कया
जा सकता है
Solution for Question 99 Ans Key: 3

eng Quantity I-
Then 2(A+B+C)=3+4+6=>A+B+C=6.5 then A=6.5-4=2.5
Time taken by A to complete the work =180/2.5=72days
Quantity II- A+B can do the total work in 60days.
They start work together for 20days ,remaining work of A+B of 40 day completed by B in 100
days therefore work of 60days of A+B will be completed by B in = =150day.

Then one day work of A will be 5-2=3 unit. Then time taken by A to complete the total work =300/3=100days.
hi मा ा I-

तब 2(A+B+C)=3+4+6=>A+B+C=6.5 अतः A=6.5-4= 2.5


काय पूरा करने के िलए A ारा िलया गया समय =180/2.5 =72 िदन
मा ा II- A+B कुल काय 60 िदन म कर सकते है।
वे एक साथ 20 िदन काम करते ह, A+B के शेष 40 िदन का काम B ारा 100 िदन म पूरा िकया जाता है इसिलए
A+B के 60 िदनो के काय को को B ारा करने म लगा समय= =150day.

तब A का एक िदन का काम 5-2 = 3 यूिनट होगा। अतःA ारा कुल काय पूरा करने म लगने वाला गया समय
= 300/3 =100 िदन

Q­100 Find the correct relationship between two quantities.
eng
Quantity I­ Find the distance, if a man goes to a distance  with speed of 60km/h reaches 20 min early, and when he goes same distance with speed of 40
km/h he reaches 20 min late?
Quantity II­ Two buses start travelling in same direction at 6:00 am with the speed of 45km/h and 60km/h. After 4hours faster bus get punctured and
takes 3 hours to repair.  Find three­fourth of 360% of the distance between these two buses at 6:00 pm if faster bus started after repairing with increasing
its speed by 10% .

01. Quantity I > Quantity II 02. Quantity I ≥  Quantity II


03. Quantity II > Quantity I 04. Quantity II ≥ Quantity I
05. Quantity I = Quantity II or Relation cannot
be established
Q­100 द ो मा ा के बीच सही सं ब ं ध थािपत क िजये ।
hi
मा ा I­ दूरी ात कर , य द कोई ि  60  कमी/घंटा क गित के साथ कसी िनि त दूरी पर जाता है, तो वह 20 िमनट पहले प च ं ता है, और जब वह 40  कमी / घंटा
क गित के साथ समान दूरी पर जाता है, तो वह 20 िमनट देरी से प च
ं ता है?
मा ा II­ दो बस सुबह 6:00 बजे 45  कमी/घंटा और 60  कमी/घंटा क गित से एक ही दशा म या ा करना शु करती ह। 4 घंटे के बाद तेज़ बस पं चर हो जाती है
और इसके मर मत म 3 घंटे लगते ह। 6:00 बजे इन दोन बस के बीच क दूरी के  360% का तीन­चौथाई ात क िजये य द तेज बस मर मत के बाद अपनी गित 10%
से बढ़ा कर या ा ारंभ करती है  ?

01. मा ा I > मा ा II 02. मा ा I ≥ मा ा II


03. मा ा II > मा ा I 04. मा ा II ≥ मा ा I
05. मा ा I = मा ा II या संबध
ं थािपत नह कया
जा सकता है
Solution for Question 100 Ans Key: 3
eng
Quantity I- => D= 80km
Quantity II- distance travelled by a bus with speed of 45km/h in 12 hours is 45×12=540km. while distance travelled by another bus
=60×4 + 66×5=570km
Required distance= of 360% of (570-540) =81km.

hi
hi
मा ा I- => D= 80 िकमी
मा ा II- 12 घंटे म 45 िकमी/घंटा क गित से बस ारा तय क गई दूरी 45 × 12 = 540 िकमी है। जबिक दूसरी बस ारा तय क गई दूरी = 60 × 4 + 66 × 5 =
570 िकमी
आव यक दूरी= of 360% of (570-540) =81िकमी

Passage for Question 101, 102, 103, 104, 105 (ST­2041SBIPOMAINS(46­50M)2019)
eng Passage

Study the data carefully and answer the following questions:

The following data is regarding the number of three different kind of sweets A, B and C made by a four confectioner Vashundhara, Sonu, Sweety and Govardhan.
The average number of sweets made by the confectioner Vashundhara is 100. The number of total sweets made by confectioner Sweety and Govardhan are equal
to the total  number of sweets made by Vashundhara and Sonu. The ratio of number of sweets A, B and C made by Sweety and Govardhan are 5: 6: 9 and 4: 1: 3
respectively. In Sonu confectionery the number of sweet A made  is half of the number of sweet C made by it which is equal to the number of sweet B made by
confectioner Vashundhara. The ratio of number of sweet A, B and C made by confectioner Vashundhara is 15: 8: 7 and total number of all three sweets made by
four confectioners is 1400.

hi Passage

आं क ड़ क ा यानपू व क अ ययन कर औ र िन िलिखत के उ र द :

िन िलिखत आंकड़े चार क फे शनर (िम ान दुकान) वसुध


ं रा, सोनू,  वीटी और गोवधन ारा बनाई गई तीन अलग­अलग कार क िमठाइय A, B और C क सं या के बारे म
ह।
वसुध
ं रा ारा बनाई गई िमठाइय क औसत सं या 100 है। क फे शनर वीटी और गोवधन ारा बनाई गई कुल िमठाइय क सं या वसुध ं रा और सोनू ारा बनाई गई िमठाइय
क कुल सं या के बराबर है। वीटी और गोवधन ारा बनयी गई िमठाई A, B और C क सं या का अनुपात मशः 5: 6: 9 और 4: 1: 3 है। सोनू क फे शनरी म िमठाई A क
मा ा उसके ारा बनाई गई िमठाई C क सं या का आधा है जो क फे शनर वसुध ं रा ारा बनाई गई िमठाई B क सं या के बराबर है। क फे शनर वसुध
ं रा ारा बनाई गई
िमठाई A, B और C क सं या का अनुपात 15: 8: 7 और चार क फे शनस ारा बनाई गई तीन िमठाइय क कुल सं या 1400 है।

Q­101 What is the resepective ratio between the total number of sweet made by confectioner Vashundhara and Sonu sweet each category ?
eng

01. 19 : 36 : 18 02. 18 : 36 : 19


03. 36 : 19 :18 04. 19 : 18 : 36
05. 19 : 36 : 18
Q­101 वसुध
ं रा और सोनू िमठाई ारा बनाई गई येक कार क कुल िमठाई क सं या के बीच मागत अनुपात या है ?
hi

01. 19 : 36 : 18 02. 18 : 36 : 19


03. 36 : 19 :18 04. 19 : 18 : 36
05. 19 : 36 : 18
Solution for Question 101 Ans Key: 1
eng

Required Ratio = (150 +40 ) : ( 80 + 280 ) : ( 70 +80)


= 190 : 360 : 180 = 19 : 36 : 18

hi

आव यक अनुपात = (150 +40 ) : ( 80 + 280 ) : ( 70 +80)


= 190 : 360 : 180 = 19 : 36 : 18

Q­102 5 If the number of total sweets made by confectioner Sweety and Govardhan are equal  and the total  number of sweets made by Vashundhara and Sonu
eng are equal then which type of sweet made maximum and how much?

01. A , 430 02. A , 490


03. B, 490 04. B, 430
05. Can't Determined
Q­102 य द क फे शनर वीटी और गोवधन ारा बनाई गई कुल िमठाइय क सं या बराबर है और वसुध
ं रा और सोनू ारा बनाई गई िमठाइय क कुल सं या बराबर है तो
hi कस कार क िमठाई अिधकतम और कतनी है?

01. A , 430 02. A , 490


03. B, 490 04. B, 430
05. िनधा रत नह कया जा सकता
Solution for Question 102 Ans Key: 2
eng

hi

Q­103 What is the average number of Sweet made by all confectionar of Sweet type C? It is given that  confectioner Sweety and Vashundhara made equal
eng number sweet each type.

01. 90 02. 95.25


03. 80 04. 85
05. None of these
Q­103 सभी क फे शनर ारा बनाई गई िमठाई "C"  कार क औसत सं या या है? यह दया गया है क क फे शनर वीटी और वसुध
ं रा ने येक कार क समान सं या म
hi िमठाई बनाई ।

01. 90 02. 95.25


03. 80 04. 85
05. इनम से कोई नह
Solution for Question 103 Ans Key: 5
eng

Rquired Average = 370 ÷ 4 = 92.5

hi

अभी औसत = 370 ÷ 4 = 92.5

Q­104 If the respective ratio of the total number of sweet made by confectioner Sweety and Goverdhan are 5 : 2 then the total number of sweet made by Sonu is
eng how much percent more or less than the total number of  sweet made by  confectioner Sweety?

01. 20% 02. 25%


03. 15% 04. Can't Determined
05. None of these
Q­104 य द क फे शनर वीटी और गोवधन ारा बनाई गई िमठाई क कुल सं या का मागत अनुपात 5 : 2 है तो सोनू ारा बनाई गई िमठाई क कुल सं या ,  वीटी ारा
hi बनाई गई िमठाई क कुल सं या से कतनी ितशत अिधक या कम है ?

01. 20% 02. 25%


03. 15% 04. िनधा रत नह कया जा सकता
05. इनम से कोई नह
Solution for Question 104 Ans Key: 1
eng

Sweety : Govardhan = 5 : 2
Sweety = =500
Sonu = 400 
Hence,
Required percent =

hi

वीटी : गोवधन = 5 : 2
वीटी = =500
सोनू = 400 
अत: 
आव यक  ितशत =

Q­105 If the number of sweet B type made by confectioner Govardhan is equal to the number of sweet made by confectioner Sonu of Sweet A type then what is
eng respective ratio of the the total number of sweet made by confectioner Sonu and Sweety of sweet B type to the total number of sweet made by
confectioner Govardhan and Vashundhara same type?

01. 15 : 53 02. 53 : 15


03. 7 : 8 04. Can't Determined
05. None of these
Q­105 य द क फे शनर गोवधन ारा बनाई गई िमठाई B  कार क सं या, क फे शनर सोनू ारा बनाई गई A  कार क िमठाई के बराबर है तो क फे शनर सोनू और वीटी
hi ारा बनाई गयी कुल B  कार क िमठाई क सं या से, क फे शनर गोवधन और वसुध
ं रा ारा बनाई गई इसी कर क िमठाई क सं या का अनुपात या है?

01. 15 : 53 02. 53 : 15


03. 7 : 8 04. िनधा रत नह कया जा सकता
05. इनम से कोई नह
Solution for Question 105 Ans Key: 2

eng

Y = 40
Govardhan's Sweet = 160 , 40 . 120
Sweety's Sweet = 20x = (700 – 320) = 480
= 120 , 144, 216
Rquired Ratio = (280+144) : (80+40) = 424 : 120 = 106 : 30 = 53 : 15

hi
Y = 40
गोवधन क िमठाइयाँ = 160 , 40 . 120
वीटी क िमठाइयाँ Sweet = 20x = (700 – 320) = 480
= 120 , 144, 216
अभी अनुपात = (280+144) : (80+40) = 424 : 120 = 106 : 30 = 53 : 15

Passage for Question 106, 107, 108, 109, 110 (ST­10 SBICLERKMAINS(41­45))
eng Passage

Study the following graph and answer the following question.
Profit earned by three companies over the years (Rs. in crores)

hi Passage

िन ाफ को यानपूवक पढ़े और दए गए का उ र दे |


तीन िविभ कंपिनय ारा ा लाभ (कर ड़ म )

Q­106 Find the average profit earned by company B in all the years.
eng

01. 52 lakh 02. 54 lakh


03. 55 lakh 04. 56 lakh
05. 58 lakh
Q­106 कंपनी B का सभी वष म िमलकर औसत लाभ ात क िजए |
hi

01. 52 लाख 02. 54 लाख


03. 55 लाख 04. 56 लाख
05. 58 लाख
Solution for Question 106 Ans Key: 2
eng average profit=270/5=54 lakh
hi औसत लाभ =270/5=54 लाख

Q­107 Which company has lowest profit % in all the years together.
eng

01. A 02. B
03. C 04. All are same
05. Can't be determined
Q­107 सभी वष म िमलाकर कस क पनी का लाभ % सबसे अिधक है ?
hi

01. A 02. B
03. C 04. सभी का समान है |
05. तय नह कर सकते |
Solution for Question 107 Ans Key: 5
eng because expenditure is not given
hi य क य नह दया गया है |

Q­108 If company A invests 16 crore rs .in the year 2013 . Find the profit % of that company in the same year.
eng

01. 33% 02. 35%


03. 40% 04. 50%
05. data inadequate
Q­108 य द कंपनी A ने वष 2013 म 16 करोड़ िनवेश कया तो उसी कंपनी का उसी वष लाभ %  ात क िजए |
hi

01. 33% 02. 35%


03. 40% 04. 50%
05. आंकड़े अधूरे है |
Solution for Question 108 Ans Key: 3
eng
profit%=

hi
लाभ %=

Q­109 The profit earned by company C in 2014 is what % the total profit earned by company A in all the years.
eng

01. 19 02. 24
03. 21 04. 32
05. 27
Q­109 वष 2014 म कंपनी C का लाभ सभी वष म कंपनी A के कुल लाभ का कतना % है ?
hi

01. 19 02. 24
03. 21 04. 32
05. 27
Solution for Question 109 Ans Key: 5
eng
required %=

hi
अभी   %=

Q­110 Find the ratio between total profit of company A and B in 2011 to the total profit of company B and C in  2014.
eng

01. 4:3 02. 3:4


03. 3:5 04. 10:9
05. 9:10
Q­110 वष 2011 म कंपनी A और B के कुल लाभ और वष 2014 म क पनी B और कंपनी C के कुल लाभ के बीच का अनुपात ात क िजए |
hi

01. 4:3 02. 3:4


03. 3:5 04. 10:9
05. 9:10
Solution for Question 110 Ans Key: 5
eng required ratio=90:100=9:10
hi अभी अनुपात =90:100=9:10

Passage for Question 111, 112, 113, 114, 115 (ST­2031 SBIPO MAINS M (21­25) 2018)
eng Passage

Read the following information carefully and answer the following questions carefully.
The graphs below shows percentage of students (Boys + Girls) appeared from different cities in class X and XII examination. 
The Table shows the ratio between the boys and girls from different cities.
                  Percentage of students X

Percentage of students XII

hi Passage

िन िलिखत जानकारी को यानपूवक पढ़ और पूछे गए का उ र द।


नीचे दए गए ाफ िविभ शहर से क ा X और XII परी ा म सि मिलत िव ा थयो (लड़क  + लड़ कयां) का ितशत दशाता है। तािलका िविभ शहर से लड़क और
लड़ कय के बीच अनुपात दशाती है।
िव ा थ यो क ा ितशत X म
िव ा थयो का ितशत XII म

Q­111 If the total number of students appeared in class X exam from all the places is 66000 then find the average number of boys appeared from cities Mumbai,
eng Chennai and Delhi in class X examination?

01. 5395 02. 6172


03. 5120 04. 5245
05. 5025
Q­111 य द सभी थान से क ा X क परी ा म सि मिलत छा क कुल सं या 66000 है तो क ा X क परी ा म मुब
ं ई, चे ई और द ली शहर से सि मिलत लड़क क
hi औसत सं या ात क िजये।

01. 5395 02. 6172


03. 5120 04. 5245
05. 5025
Solution for Question 111 Ans Key: 4

eng Number of boys appeared from Mumbai

Number of boys appeared from Chennai

Number of boys appeared from Delhi

Required average =  

hi मु बई से सि मिलत लड़क क कुल सं या


चे नई से सि मिलत लड़क क कुल सं या

िद ली से सि मिलत लड़क क कुल सं या

अभी औसत =  

Q­112 If the difference between the number of boys and girls appeared from Hyderabad in class XII exam is 150 then find the number of girls students appeared
eng from Mumbai in class XII examination.

01. 420 02. 172


03. 120 04. 300
05. 500
Q­112 य द क ा XII म हैदराबाद से लड़क और लड़ कय क सं या के बीच अंतर 150 है तो XII क परी ा म मुब
ं ई से सि मिलत लड़ कय क सं या ात क िजये।
hi

01. 420 02. 172


03. 120 04. 300
05. 500
Solution for Question 112 Ans Key: 1

eng
Number of students appeared from Delhi = 

Number of girls appeared from Delhi = 

hi
िद ली से सि मिलत िव ािथयो क कुल सं या  = 

िद ली से सि मिलत लड़िकय क कुल सं या  = 

Q­113 Out of total students appeared from Chennai in class XII, 60% of the boys passed which is equal to 288 then find number of boys appeared from Delhi in
eng class XII exam.

01. 856 02. 1089


03. 1200 04. 1300
05. 1500
Q­113 XII व क ा म चे ई से सि मिलत कुल िव ा थयो म से, 60% लड़के पास ए जो क 288 के बराबर है, तो क ा XII म द ली से सि मिलत लड़क क सं या ात
hi क िजये।

01. 856 02. 1089


03. 1200 04. 1300
05. 1500
Solution for Question 113 Ans Key: 2

eng Total number of boys appeared from Chennai= 288×100/60= 480
Number of girls appeared from Chennai= 480×7/4= 840
Total number of students appeared from Chennai= 480+840= 1320
Number of students appeared from Delhi= 1320×22/16= 1815
Number of boys appeared from Delhi= 1815×3/5= 1089
hi चे नई से सि मिलत लड़क क कुल सं या = 288 × 100/60 = 480
चे नई से सि मिलत लड़िकय क कुल सं या = 480 × 7/4 = 840
चे नई से सि मिलत िव ािथयो क कुल सं या = 480 + 840 = 1320
िद ली से सि मिलत िव ािथयो क कुल सं या = 1320 × 22/16 = 1815
िद ली से सि मिलत लड़क क कुल सं या = 1815 × 3/5 = 1089
Q­114 In class X exam, 280 girls appeared from Mumbai then find the average number of students appeared from Kolkata and Chennai in class X exam.
eng

01. 395 02. 172


03. 220 04. 300
05. 500
Q­114 क ा X क परी ा म, 280 लड़ कयां मुब
ं ई से सि मिलत ई ,तो क ा X परी ा म कोलकाता और चे ई से सि मिलत िव ा थयो क औसत सं या ात क िजये।
hi

01. 395 02. 172


03. 220 04. 300
05. 500
Solution for Question 114 Ans Key: 3
eng Total number students appeared from Mumbai= 280×11/7= 440
Number of students appeared from Kolkata= 440×10/25= 176
Number of students appeared from Chennai= 440×15/25= 264
Required average= (176+264)/2 = 440/2 = 220 students
hi मुब
ं ई से सि मिलत िव ा थयो क कुल सं या= 280 × 11/7 = 440 
कोलकाता से सि मिलत िव ा थयो क कुल सं या = 440 × 10/25 = 176
चे ई से सि मिलत िव ा थयो क कुल सं या = 440 × 15/25 = 264 
अभी औसत = (176 + 264) / 2 = 440/2 = 220 िव ाथ

Q­115 The average number of boys appeared from Delhi in Class X and XII examination is 225 and the number of boys appeared in class XII is 20% less than
eng the number of boys appeared in Class X then find the number of girls appeared from Delhi in class X.

01. 375 02. 172


03. 120 04. 300
05. 500
Q­115 क ा X और XII परी ा म द ली से सि मिलत लड़क क औसत सं या 225 है और क ा XII म सि मिलत लड़क क सं या क ा X म सि मिलत लड़क क सं या से
hi 20% कम है, तो क ा X म द ली से सि मिलत लड़ कय क सं या ात क िजये।

01. 375 02. 172


03. 120 04. 300
05. 500
Solution for Question 115 Ans Key: 1
eng Total boys appeared from Delhi in class X and XII
= 225×2 = 450
Number of boys appeared in class X=

Number of girls appeared from Delhi in Class X = 

hi क ा X और XII म िद ली से सि मिलत कुल लड़क क सं या = 225 × 2 = 450

क ा X म सि मिलत िव ािथयो क सं या =

क ा X म िद ली से सि मिलत लड़िकय क सं या =

Q­116 Who among the following is the author of the book titled "Unfinished"?
eng

01. Chetan Bhagat 02. Salman Rushdie


03. Priyanka Chopra 04. Anuja Chauhan
05. Salman Khan
Q­116 िन िलिखत म से कौन "अन फिन ड" नामक पु तक के लेखक ह?
hi

01. चेतन भगत 02. सलमान दी


03. ि यंका चोपड़ा 04. अनुजा चौहान
05. सलमान खान
Solution for Question 116 Ans Key: 3
Solution for Question 116 Ans Key: 3
eng "Unfinished" is a 2021 memoir of global icon Priyanka Chopra. It has been described as a "collection of personal essays, stories and observations" and
chronicles important moments in Chopra's life and her twenty­year­long career.
hi "अन फिन ड" वैि क आइकन ि यंका चोपड़ा का 2021 का सं मरण है। इसे " ि गत िनबंध , कहािनय और ट पिणय के सं ह" के प म व णत कया गया है और
चोपड़ा के जीवन और उनके बीस साल के लंबे क रयर म मह वपूण ण का वणन करता है।

Q­117 The Industrial Development Bank of India was set up as a wholly owned subsidiary of the______.
eng

01. RBI 02. SBI


03. SEBI 04. IRDA
05. TRAI
Q­117 भारतीय औ ोिगक िवकास बक को __क पूण वािम व वाली सहायक कंपनी के प म थािपत कया गया था।
hi

01. भारतीय रजव बक 02. भारतीय टेट बक


03. सेबी 04. आईआरडीए
05. ाई
Solution for Question 117 Ans Key: 1
eng The Industrial Development Bank of India was set up as a wholly owned subsidiary of the RBI on July 1st 1964 under an act of parliament.
hi 1 जुलाई 1964 को संसद के एक अिधिनयम के तहत भारतीय औ ोिगक िवकास बक को आरबीआई क पूण वािम व वाली सहायक कंपनी के प म थािपत कया
गया था।

Q­118 Recently how much amount the Bangladesh has signed agreement with ADB for two loans?
eng

01. USD 292 million 02. USD 392 million


03. USD 492 million 04. USD 592 million
05. USD 692 million
Q­118 हाल ही म बां लादेश ने एडीबी के साथ दो ऋण के िलए कतनी रािश के समझौते पर ह ता र कए ह?
hi

01. 292 िमिलयन अमरीक डालर 02. 392 िमिलयन अमरीक डालर


03. 492 िमिलयन अमरीक डालर 04. 592 िमिलयन अमरीक डालर
05. 692 िमिलयन अमरीक डालर
Solution for Question 118 Ans Key: 1
eng The Government of Bangladesh signed two separate agreements with Asian Development Bank (ADB) for loans worth USD 292 million on 1 March
2022.
hi बां लादेश सरकार ने 1 माच 2022 को 292 िमिलयन अमरीक डालर के ऋण के िलए एिशयाई िवकास बक (एडीबी) के साथ दो अलग­अलग समझौत पर ह ता र
कए।

Q­119 State Bank of India was earlier known as
eng

01. Bank of Hindustan 02. Imperial Bank of India


03. Bank of Calcutta 04. Bank of Maharashtra
05. Bank of Bombay
Q­119 भारतीय टेट बक को पहले कस नाम से जाना जाता था?
hi

01. बक ऑफ हंद ु तान 02. इंपी रयल बक ऑफ इंिडया


03. बक ऑफ कलक ा 04. बक ऑफ महारा
05. बक ऑफ बॉ बे
Solution for Question 119 Ans Key: 2

eng State Bank of India was earlier known as 'Imperial Bank of India'.
hi भारतीय टेट बक को पहले 'इंपी रयल बक ऑफ इंिडया' के प म जाना जाता था।

Q­120 Recently EU Commission has announced temporary residence permits to refugees and allows them to work in the how many nations?
eng

01. 30 02. 28
03. 27 04. 22
05. 15
Q­120 हाल ही म यूरोपीय संघ आयोग ने शरणा थय को अ थायी िनवास परिमट क घोषणा क है और उ ह कतने देश म काम करने क अनुमित दी है?
hi

01. 30 02. 28
03. 27 04. 22
05. 15
Solution for Question 120 Ans Key: 3

eng EU Commission has announced temporary residence permits to refugees and allows them to work in 27­nation bloc.
hi यूरोपीय संघ आयोग ने शरणा थय के िलए अ थायी िनवास परिमट क घोषणा क है और उ ह 27­रा लॉक म काम करने क अनुमित दी है।

Q­121 Who among the following has becomes world's No.1 Test all­rounder in latest ICC rankings?
eng

01. Ravindra Jadeja 02. Shakib Al Hasan


03. Kyle Jamieson 04. Chris Woakes
05. Virat Kohali
Q­121 िन िलिखत म से कौन नवीनतम ICC र कंग म दुिनया का नंबर 1 टे ट ऑलराउंडर बन गया है?
hi

01. रव जडेजा 02. शा कब अल हसन


03. काइल जैमीसन 04. स वो स
05. िवराट कोहली
Solution for Question 121 Ans Key: 1
eng India's Ravindra Jadeja becomes world's No.1 Test all­rounder in latest ICC rankings.
hi भारत के रव जडेजा नवीनतम ICC र कंग म दुिनया के नंबर 1 टे ट ऑलराउंडर बन गए ह।

Q­122 The currency note of one rupee bear the signature of who among the following?
eng

01. President of India 02. Prime Minister of India


03. Finance Secretary of India 04. Governor of RBI
05. CAG of India
Q­122 एक पये के करसी नोट पर िन िलिखत म से कौन ह ता र करता है?
hi
01. भारत के रा पित 02. भारत के धानमं ी
03. भारत के िव सिचव 04. आरबीआई के गवनर
05. भारत के िनयं क एवं महालेखा परी क
Solution for Question 122 Ans Key: 3
eng The One Rupee note is issued by Ministry of Finance and it bears the signatures of Finance Secretary, while other notes bear the signature of Governor
RBI.
hi वन पी नोट िव मं ालय ारा जारी कया जाता है और इसम िव सिचव के ह ता र होते ह, जब क अ य नोट गवनर RBI के ह ता र होते ह।

Q­123 Which state has launched "Red Light On, Gaadi Off" campaign across 100 crossings in the city to reduce vehicular emissions?
eng

01. Haryana 02. Uttar Pradesh


03. Tamil Nadu 04. Assam
05. Delhi
Q­123 कस रा य ने वाहन के उ सजन को कम करने के िलए शहर म 100  ॉ संग पर "रेड लाइट ऑन, गाड़ी ऑफ" अिभयान शु कया है?
hi

01. ह रयाणा 02. उ र देश


03. तिमलनाडु 04. असम
05. द ली
Solution for Question 123 Ans Key: 5
eng The Delhi government launched the "Red Light On, Gaadi Off" campaign across 100 crossings in the city, noting that if people switch off engines at traffic
signals, pollution can be cut down by 13­20 per cent.
hi द ली सरकार ने शहर म 100  ॉ संग पर "रेड लाइट ऑन, गाड़ी ऑफ" अिभयान शु कया, यह देखते ए क अगर लोग ै फक िस ल पर इंजन बंद कर देते ह, तो
दूषण म 13­20  ितशत क कटौती क जा सकती है।

Q­124 Which of the following state has retained the top slot on "the logistics index chart"?
eng

01. Assam 02. Gujarat


03. Maharashtra 04. Tamil Nadu
05. Kerala
Q­124 िन िलिखत म से कस रा य ने "लॉिजि ट स इं डे स चाट" पर शीष थान बरकरार रखा है?
hi

01. असम 02. गुजरात


03. महारा 04. तिमलनाडु
05. केरल
Solution for Question 124 Ans Key: 2

eng Gujarat has retained the top slot on "the logistics index chart", an indicator of the efficiency of logistical services necessary for promoting exports and
economic growth, according to a Commerce Ministry report released 8 November 2021.
hi वािण य मं ालय क  8 नवंबर 2021 को जारी एक रपोट के अनुसार, गुजरात ने "लॉिजि ट स इं डे स चाट" पर शीष थान बरकरार रखा है, जो िनयात और आ थक
िवकास को बढ़ावा देने के िलए आव यक लॉिजि टक सेवा क द ता का संकत े क है।

Q­125 Which of the following state has becomes ninth state to withdraw general consent to CBI for investigation?
eng

01. Kerala 02. Meghalaya


03. Jharkhand 04. Assam
05. Uttar Pradesh
Q­125 िन िलिखत म से कौन सा रा य जांच के िलए सीबीआई से सामा य सहमित वापस लेने वाला नौवां रा य बन गया है?
hi

01. केरल 02. मेघालय


03. झारखंड 04. असम
05. उ र देश
Solution for Question 125 Ans Key: 2
eng The NDA-ruled Meghalaya has become the ninth state to withdraw the general consent to the CBI for investigation.
hi एनडीए शािसत मेघालय जांच के िलए सीबीआई क आम सहमित वापस लेने वाला नौवां रा य बन गया है।

Q­126 Which of the following reflects the operational position of the firm during a particular period?
eng

01. Balance Sheet 02. Bank statement


03. Income statement 04. Loan statement
05. None of these
Q­126 इनम से कौन सा कसी िवशेष अविध के दौरान फम क प रचालन ि थित को दशाता है?
hi

01. तुलन प 02. बक िववरण


03. आय िववरण 04. ऋण िववरण
05. इनमे से कोई नह
Solution for Question 126 Ans Key: 3
eng Income statement is also called as profit and loss account, which reflects the operational position of the firm during a particular period.
hi आय िववरण को लाभ और हािन खाता भी कहा जाता है, जो कसी िवशेष अविध के दौरान फम क प रचालन ि थित को दशाता है।

Q­127 Which of the following organisation has hold the 7th "National Conference on Economics of Competition Law"?
eng

01. Securities and Exchange Board of India 02. Reserve Bank of India


03. Finance Commission 04. Competition Commission of India
05. World Trade Organization
Q­127 िन िलिखत म से कस संगठन ने 7 वां " ित पधा कानून के अथशा पर रा ीय स मेलन" आयोिजत कया है?
hi
01. भारतीय ितभूित और िविनमय बोड 02. भारतीय रजव बक
03. िव आयोग 04. भारतीय ित पधा आयोग
05. िव ापार संगठन
Solution for Question 127 Ans Key: 4
eng The Competition Commission of India (CCI) has hold the 7th National Conference on Economics of Competition Law, March 4, 2022 in Virtual Mode.
hi भारतीय ित पधा आयोग (CCI) ने वचुअल मोड म 4 माच, 2022 को ित पधा कानून के अथशा पर 7 वां रा ीय स मेलन आयोिजत कया है।

Q­128 The concept of mutual funds was found in which country?
eng

01. China 02. Japan


03. India 04. Brazil
05. Belgium
Q­128 यूचुअल फंड क अवधारणा कस देश म िवकिसत ई थी?
hi

01. चीन 02. जापान


03. भारत 04. ाजील
05. बेि जयम
Solution for Question 128 Ans Key: 5
eng In the year 1822, the concept of mutual funds was found in Belgium.
hi वष 1822 म बेि जयम म यूचुअल फंड क अवधारणा का िवकास आ था

Q­129 Which of the following is a swap?
eng

01. Interest Rate Swap 02. Currency Swap


03. Credit Default Swap 04. All of these
05. None of these
Q­129 इनम से कौन सा वैप है?
hi

01. याज दर वैप 02. करसी वैप


03. िे डट िडफ़ॉ ट वैप 04. ये सभी
05. इनमे से कोई नह
Solution for Question 129 Ans Key: 4
eng Interest Rate Swap, Currency Swap & Credit Default Swap are all types of Swap.
hi याज दर वैप, मु ा वैप और िे डट िडफ़ॉ ट वैप सभी कार के वैप ह।

Q­130 Where Sourav Ganguly has laid the foundation stone of the world's third largest cricket stadium?
eng

01. Bhopal 02. Hyderabad


03. Jaipur 04. Lucknow
05. Chennai
Q­130 सौरव गांगल
ु ी ने दुिनया के तीसरे सबसे बड़े केट टेिडयम क आधारिशला कहाँ रखी है?
hi

01. भोपाल 02. हैदराबाद


03. जयपुर 04. लखनऊ
05. चे ई
Solution for Question 130 Ans Key: 3
eng Rajasthan Chief Minister, Ashok Gehlot and BCCI President Sourav Ganguly laid the foundation stone of the world's third­largest cricket stadium in Jaipur.
The International Cricket Stadium in Jaipur will be the second­largest cricket stadium in India and the third­largest in the world.
hi राज थान के मु यमं ी अशोक गहलोत और बीसीसीआई अ य सौरव गांगल ु ी ने जयपुर म दुिनया के तीसरे सबसे बड़े केट टेिडयम क आधारिशला रखी। जयपुर के
अंतरा ीय केट टेिडयम म भारत का दूसरा सबसे बड़ा और दुिनया का तीसरा सबसे बड़ा केट टेिडयम है।

Q­131 Where is the Headquarters of World Intellectual Property Organisation (WIPO)?
eng
01. Vienna, Austria 02. Rome, Italy
03. Paris, France 04. Geneva, Switzerland
05. Ottawa, Canada
Q­131 िव बौि क संपदा संगठन (ड यूआईपीओ) का मु यालय कहां है?
hi

01. िवयना, ऑि या 02. रोम, इटली


03. पे रस,  ांस 04. िजनेवा, ि व ज़रलड
05. ओटावा, कनाडा
Solution for Question 131 Ans Key: 4
eng Geneva, Switzerland is the Headquarters of World Intellectual Property Organisation (WIPO).
hi िव बौि क संपदा संगठन (ड यूआईपीओ) का मु यालय िजनेवा, ि व ज़रलड म है.

Q­132 What is the rank of India in the "Wealth Report 2022" released by Knight Frank?
eng

01. 8th 02. 4th


03. 6th 04. 3rd
05. 7th
Q­132 नाइट क ारा जारी "वे थ रपोट 2022" म भारत का र क या है?
hi

01. 8 व 02. 4 व


03. 6 व 04. 3 व
05. 7 व
Solution for Question 132 Ans Key: 4
eng India is ranked 3rd in terms of most number of billionaires' population globally in 2021, as per Knight Frank's latest edition of The "Wealth Report 2022".
hi नाइट क के  "वे थ रपोट 2022" के नवीनतम सं करण के अनुसार, 2021 म िव तर पर अरबपितय क आबादी क सबसे अिधक सं या के मामले म भारत तीसरे
थान पर है।

Q­133 What is the name of the official mobile app of the Parliament of India, launched by Lok Sabha Speaker Om Birla?
eng

01. Digital Loksabha App 02. Digital Diet App


03. Digital Parliament App 04. Digital House App
05. Digital Sansad App
Q­133 लोकसभा अ य ओम िबरला ारा लॉ च कए गए भारत क संसद के आिधका रक मोबाइल ऐप का नाम या है?
hi

01. िडिजटल लोकसभा ऐप 02. िडिजटल डाइट ऐप


03. िडिजटल पा लयामट ऐप 04. िडिजटल हाउस ऐप
05. िडिजटल संसद ऐप
Solution for Question 133 Ans Key: 5
eng The Lok Sabha Speaker Om Birla launched the official mobile application of the Parliament named 'Digital Sansad App', to allow citizens to access live
proceedings of the House, including the Union Budget 2022 live.
hi लोकसभा अ य ओम िबरला ने संसद के आिधका रक मोबाइल एि लकेशन को 'िडिजटल संसद ऐप' नाम से लॉ च कया, िजससे नाग रक को क ीय बजट 2022
सिहत सदन क लाइव कायवाही तक प चं ने क अनुमित िमल सके।

Q­134 What is the full form of ‘P­Notes’?
eng

01. Permanent Notes 02. Purchase Notes


03. Participatory Notes 04. Private Notes
05. None of these
Q­134 'P­Notes' का पूण प या है?
hi

01. Permanent Notes 02. Purchase Notes


03. Participatory Notes 04. Private Notes
05. इनम से कोई नह
Solution for Question 134 Ans Key: 3
eng Participatory Notes, commonly known as P­Notes or PNs are instruments issued by registered Foreign Institutional Investors (FII) to overseas investors.
hi Participatory Notes (पा टिसपेटरी नो स) को सामा यतः P­Notes या PN's के प म जाना जाता है जो िवदेशी िनवेशक को पंजीकृत िवदेशी सं थागत िनवेशक
(एफआईआई)  ारा जारी कए जाने वाले साधन ह।

Q­135 What do you understand by the term ‘Mortgage’?
eng

01. Sale of moveable security in the event of 02. Registration of charge with the Registrar of Companies


default by the borrower
03. Making the security of immovable property 04. Registration of charge with the Regional Transport Authority
available as a cover for a home loan by the
borrower
05. Returning of the security to borrower by the
bank of receipt of full payment
Q­135 'बंधक' श द से आप या समझते ह?
hi

01. उधारकता ारा िडफ़ॉ ट क ि थित म चल 02. कंपिनय के रिज ार के साथ भार का पंजीकरण
सुर ा क िब
03. उधारकता ारा होम लोन के िलए अचल संपि 04. े ीय प रवहन ािधकरण के साथ पदभार का पंजीकरण
क ितभूित जमानत उपल ध कराना
05. पूण भुगतान क रसीद के बक ारा उधारकता
को सुर ा वापस करना
Solution for Question 135 Ans Key: 3

eng Mortgage­ a legal agreement by which a bank, building society, etc. lends money at interest in exchange for taking title of the debtor's property, with the condition that the


conveyance of title becomes void upon the payment of the debt.

hi बंधक­ एक कानूनी समझौता, िजसके ारा एक बक, भवन िनमाण समाज आ द, ऋणी क संपि का शीषक लेने के बदले म याज पर पैसा उधार देता है, इस शत के
साथ क ऋण के भुगतान पर शीषक का उ लेख शू य हो जाता है।

Q­136 Who among the following has won the 37th "Masters Title" in Paris?
eng

01. Daniil Medvedev 02. Novak Djokovic


03. Alexander Zverev 04. Andrey Rublev
05. Rafael Nadal
Q­136 िन िलिखत म से कसने पे रस म 37वां "मा टस िखताब" जीता है?
hi

01. डेिनयल मेदवेदव


े 02. नोवाक जोकोिवच
03. अले जडर वेरव े 04. एं ी बलेव
05. राफेल नडाल
Solution for Question 136 Ans Key: 2
eng World number one Novak Djokovic won a sixth Paris title and record 37th Masters crown, beating Daniil Medvedev 4­6, 6­3, 6­3 in the final.
hi िव के नंबर एक नोवाक जोकोिवच ने फाइनल म डेिनयल मेदवेदव
े को 4­6, 6­3, 6­3 से हराकर छठा पे रस िखताब जीता और 37वां मा टस िखताब का रकॉड
बनाया।

Q­137 'Amoria thorae' which has been seen in the news is a new species of which one?
eng

01. Sheep 02. Spider


03. Turtle 04. Snake
05. Marine Snail
Q­137 'अमो रया थोरे ' जो खबर म रहा है,  कसक एक नई जाित है?
hi

01. भेड़ 02. मकड़ी


03. कछुए 04. साँप
05. समु ी घ घा
Solution for Question 137 Ans Key: 5
eng Amoria thorae, a new species of the carnivorous volute family of marine snails, was named in honor of long­time Brisbane resident Mrs. Thora Whitehead,
whose collection was recently donated to the museum. The new species is so rare, scientists have yet to see a live specimen.
hi अमो रया थोरे , समु ी घ घे के मांसाहारी िवलेय प रवार क एक नई जाित का नाम लंबे समय से ि बेन िनवासी ीमती थोरा हाइटहेड के स मान म रखा गया था,
िजसका सं ह हाल ही म सं हालय को दान कर दया गया था। नई जाित इतनी दुलभ है, वै ािनक को अभी तक एक जीिवत नमूना देखना बाक है।

Q­138 Who among the following has won the title of "Miss India Earth" 2021?
eng

01. Sonam Verma 02. Priyanka Sharma


03. Rani Soni 04. Rashmi Madhuri
05. Vibha Singh
Q­138 िन िलिखत म से कसने "िमस इंिडया अथ" 2021 का िखताब जीता है?
hi

01. सोनम वमा 02. ि यंका शमा


03. रानी सोिनक 04. रि म माधुरी
05. िवभा संह
Solution for Question 138 Ans Key: 4
eng The most awaited name of the winners of Miss Divine Beauty pageant 2021 is revealed now; Rashmi Madhuri from Bengaluru a 27yr old entrepreneur of a
pharmaceutical company was crowned the title of "Miss India Earth" 2021 by the former queen Tanvi Kharote.
hi िमस िडवाइन यूटी पेजट 2021 के िवजेता का ब तीि त नाम अब सामने आया है; बगलु क रि म माधुरी एक फामा यु टकल कंपनी क  27 वष य उ मी को पूव
रानी त वी खरोटे ारा "िमस इंिडया अथ" 2021 का िखताब दया गया।

Q­139 OPEC is a group of countries which are _________.
eng

01. exporting Oil 02. producing cotton


03. rich and developed 04. developing and poor
05. None of these
Q­139 ओपेक उन देश का एक समूह है, जो­
hi

01. तेल िनयात कर रहे ह 02. कपास का उ पादन कर रहे ह


03. धनी और िवकिसत ह 04. िवकासशील और िनधन ह
05. इनम से कोई नह
Solution for Question 139 Ans Key: 1
eng OPEC is a group of countries which are exporting Oil.
hi ओपेक तेल िनयात कर रहे देश का एक समूह है।

Q­140 Cheques that are issued by a bank and function as cash but are protected against loss or theft are called­
eng

01. Traveler’s cheque 02. Negotiable cheque


03. Crossed cheque 04. Special cheque
05. Transferable cheque
Q­140 बक ारा जारी कया जाने वाला चेक जो नकदी के प म काय करता है ले कन, हािन या चोरी के िखलाफ सुरि त होता है, उसे या कहा जाता है?
hi

01. या ी चेक 02. पर ा य चेक


03. रेखां कत चेक 04. िवशेष चेक
05. ह तांतरणीय चेक
Solution for Question 140 Ans Key: 1
eng Cheques that are issued by a bank and function as cash but are protected against loss or theft are called­'Traveler’s cheque'.
hi वे चेक जो बक और फ़ं शन ारा नकद के प म जारी कए जाते ह, ले कन नुकसान या चोरी से सुरि त रहते ह, '­ ेलर चेक' कहलाते ह।

Q­141 Civil Aviation Minister Jyotiraditya Scindia on 18 February 2022 has inaugurated direct flight between Delhi and which historic city under the "UDAN"
eng scheme?

01. Mathura 02. Khajuraho


03. Wayanad 04. Pune
05. Amravati
Q­141 नाग रक उ यन मं ी योितरा द य संिधया ने 18 फरवरी 2022 को "उड़ान" योजना के तहत द ली और कस ऐितहािसक शहर के बीच सीधी उड़ान का उ ाटन कया
hi है?
01. मथुरा 02. खजुराहो
03. वायनाड 04. पुणे
05. अमरावती
Solution for Question 141 Ans Key: 2

eng Civil Aviation Minister Jyotiraditya Scindia on 18 Febraury 2022 has inaugurated direct flight between Delhi and Khajuraho Madhya Pradesh under the
"UDAN" scheme. The flight service has been started two days before the scheduled Khajuraho festival.
hi नाग रक उ यन मं ी योितरा द य संिधया ने 18 फरवरी 2022 को "उड़ान" योजना के तहत द ली और खजुराहो म य देश के बीच सीधी उड़ान का उ ाटन कया
है। खजुराहो उ सव से दो दन पहले उड़ान सेवा शु कर दी गई है।

Q­142 The 19th edition of the India­US "Military Cooperation Group (MCG)" meeting was held at which of the following place?
eng

01. Pokhran 02. Noida


03. Pune 04. Bikaner
05. Agra
Q­142 िन िलिखत म से कस थान पर भारत­अमे रका "सै य सहयोग समूह (एमसीजी)" बैठक का 19 वां सं करण आयोिजत कया गया है?
hi

01. पोखरण 02. नोएडा


03. पुणे 04. बीकानेर
05. आगरा
Solution for Question 142 Ans Key: 5
eng The 19th edition of the India­US "Military Cooperation Group (MCG)" meeting was held in Agra, Uttar Pradesh.
hi भारत­अमे रका "सै य सहयोग समूह (एमसीजी)" बैठक का 19 वां सं करण आगरा, उ र देश म आयोिजत कया गया था।

Q­143 Subprime lending means
eng

01. making loans to people who may have 02. giving loans to retired people at concessional rates


difficulty maintaining the repayment schedule
03. lending money to students who have no 04. lending to specialized banks only
means of financing their own studies
05. All except 3
Q­143 सब­ ाइम ल डंग का अथ होता है­
hi

01. िनयिमत चुकौती न कर पाने वाले लोग को 02. सेवािनवृ लोग को रयायती दर पर ऋण दान करना
ऋण देने के िलये
03. वयं के अ ययन के िलये िव पोषण का कोई 04. िवशेषीकृत बक को उधार देना
साधन न रखने वाले छा को धन उधार देना
05. 3 के अित र सभी
Solution for Question 143 Ans Key: 1

eng These loans are characterized by higher interest rates, poor quality collateral, and less favorable terms in order to compensate for higher credit risk.
hi इस कार के ऋण क िवशेषता है­  उ याज दर, खराब गुणव ा जमानत, कम अनुकूल शत व ितपू त के िलये उ ऋण जोिखम।

Q­144 Exchange control was introduced in India under the Defence of India Rules on __.
eng

01. 11­Sep­1939 02. 21­Sep­1939


03. 13­Sep­1939 04. 30­Sep­1939
05. 3­Sep­1939
Q­144 भारत म िवदेशी मु ा िनयं ण भारत र ा कानून (DIR) __ को शु आ था।
hi

01. 11 िसतंबर 1939 02. 21 िसतंबर 1939


03. 13 िसतंबर 1939 04. 30 िसतंबर 1939
05. 03 िसतंबर 1939
Solution for Question 144 Ans Key: 5

eng Exchange control was introduced in India under the Defence of India Rules on September 3, 1939 on a temporary basis.
hi भारत म िवदेशी मु ा िनयं ण भारत र ा कानून (DIR) के अंतगत अ थाई तौर पर 3 िसतंबर 1939 को शु आ था।
hi भारत म िवदेशी मु ा िनयं ण भारत र ा कानून (DIR) के अंतगत अ थाई तौर पर 3 िसतंबर 1939 को शु आ था।

Q­145 SEBI has formed a four­member high powered advisory committee on settlement orders and compounding of offences. Who is the head of this committee?
eng

01. Rajnish Kumar 02. Dinesh Kumar Khara


03. Rajiv Kumar 04. Vijay C Daga
05. PK Malhotra
Q­145 सेबी ने िनपटान आदेश और अपराध के कं पाउं डंग पर चार सद यीय उ ािधकार ा सलाहकार सिमित का गठन कया है। इस सिमित का मुख कौन है?
hi

01. रजनीश कुमार 02. दनेश कुमार खरा


03. राजीव कुमार 04. िवजय सी डागा
05. पीके म हो ा
Solution for Question 145 Ans Key: 4

eng According to the Securities and Exchange Board of India, the committee has chaired by Vijay C Daga, retired judge of High Court of Bombay.
hi भारतीय ितभूित और िविनमय बोड के अनुसार, सिमित क अ य ता बंबई उ यायालय के सेवािनवृ यायाधीश िवजय सी डागा ने क है।

Q­146 Recently which of the following City has the two day long "Study in India (SII)" 2022 meet was inaugurated?
eng

01. Kathmandu 02. Dhaka


03. Paris 04. New Delhi
05. Moscow
Q­146 हाल ही म िन िलिखत म से कस शहर म दो दवसीय " टडी इन इंिडया (एसआईआई)" 2022 बैठक का उ ाटन कया गया है?
hi

01. काठमांडू 02. ढाका


03. पे रस 04. नई द ली
05. मा को
Solution for Question 146 Ans Key: 2

eng The two day long "Study in India (SII)" 2022 meet was inaugurated in Dhaka on 4 March 2022. The event organised by the High Commission of India
was inaugurated by the Education Minister of Bangladesh Dr. Dipu Moni and High Commissioner of India to Bangladesh Vikram Doraiswami.
hi दो दवसीय " टडी इन इंिडया (एसआईआई)" 2022 बैठक का उ ाटन 4 माच 2022 को ढाका म आ। भारतीय उ ायोग ारा आयोिजत काय म का उ ाटन
बां लादेश के िश ा मं ी डॉ. दीपू मोनी और बां लादेश म भारत के उ ायु िव म दोराई वामी ने कया है।

Q­147 Recently SMILE­ scheme launched for to provide welfare measures of which community?
eng

01. Beggars 02. Trans­gender


03. Devdasi 04. 1 and 2 only
05. 1, 2 and 3 only
Q­147 हाल ही म SMILE­ योजना कस समुदाय के क याणकारी उपाय दान करने के िलए शु क गई है?
hi

01. याचक 02. ांस­जडर


03. देवदासी 04. केवल 1 और 2
05. केवल 1, 2 और 3
Solution for Question 147 Ans Key: 4
eng "SMILE­ scheme" launched in New Delhi for provide welfare measures to Trans­genders and beggars community.
hi ांस­जडर और याचक समुदाय को क याणकारी उपाय दान करने के िलए नई द ली म "मु कान­योजना" शु क गई।

Q­148 Debt Recovery Tribunals accept petitions from banks and financial institutions, only when the amount of suit is for Rs___________.
eng

01. 100,000 and above 02. 500,000 and above


03. 750,000 and above 04. 1,000,000 and above
05. None of these
Q­148 ऋण वसूली यायािधकरण बक तथा िव ीय सं थान से तभी यािचकाएं वीकार करते है जब वाद क रािश ....................... हो।
hi

01. 02.
01. 100,000 और ऊपर 02. 500,000 और ऊपर
03. 750,000 और ऊपर 04. 1,000,000 और ऊपर
05. इनम से कोई नह
Solution for Question 148 Ans Key: 4
eng Debt Recovery Tribunals accept petitions from banks and financial institutions, only when the amount of suit is for Rs 1,000,000 and above.
hi ऋण वसूली यायािधकरण बक और िव ीय सं थान से यािचका वीकार करते ह, केवल तभी जब मुकदमा क रािश 1,000,000  पये और उससे अिधक हो ।

Q­149 Micro credit in India comes under......................
eng

01. Private Banking 02. NBFC


03. Rural Banking 04. Cooperative Banking
05. None of these
Q­149 भारत म सू म ऋण ___________के तहत आता है।
hi

01. िनजी ब कंग 02. एनबीएफसी


03. ामीण ब कंग 04. सहकारी ब कंग
05. इनम से कोई नही
Solution for Question 149 Ans Key: 2
eng NBFCs are financial institutions that provide banking services without meeting the legal definition of a bank.
hi गैर ब कंग िव ीय क पनी (एनबीएफसी) गैर ब कंग िव ीय क पनी (एनबीएफसी) एक िव ीय सं थान है जो बक क कानूनी प रभाषा के िबना ब कंग सेवाएं दान
करती है।

Q­150 National Institute of Urban Affairs (NIUA) signed MoU with which institution to collaborate on 'Sustainable Cities India program?
eng

01. Asian Development Bank 02. World Economic Forum


03. NITI Aayog 04. World Bank
05. United Nations Development Programme
Q­150 नेशनल इं टी ूट ऑफ अबन अफेयस (NIUA) ने 'स टेनब
े ल िसटीज इंिडया ो ाम' पर सहयोग करने के िलए कस सं थान के साथ समझौता ापन पर ह ता र कए?
hi

01. एिशयाई िवकास बक 02. िव आ थक मंच


03. नीित आयोग 04. िव बक
05. संयु रा िवकास काय म
Solution for Question 150 Ans Key: 2

eng The World Economic Forum and the National Institute of Urban Affairs (NIUA) signed an MoU to collaborate on the 'Sustainable Cities India program'.
hi व ड इकोनॉिमक फोरम और नेशनल इं टी ूट ऑफ अबन अफेयस (एनआईयूए) ने 'स टेनब
े ल िसटीज इंिडया ो ाम' पर सहयोग करने के िलए एक समझौता ापन
पर ह ता र कए।

Q­151 How many countries are the member of United Nation?
eng

01. 194 02. 169


03. 193 04. 189
05. 164
Q­151 संयु रा के सद य कतने देश ह?
hi

01. 194 02. 169


03. 193 04. 189
05. 164
Solution for Question 151 Ans Key: 3
eng 193 Countries are the member of United Nation.
hi संयु रा के सद य 193 देश ह.

Q­152 How many years of diplomatic relations have been completed between India and Belgium?
eng
01. 80 years 02. 75 years
03. 50 years 04. 30 years
05. 65 years
Q­152 भारत और बेि जयम के बीच राजनियक संबध
ं के कतने वष पूरे हो चुके ह?
hi

01. 80 वष 02. 75 वष


03. 50 वष 04. 30 वष
05. 65 वष
Solution for Question 152 Ans Key: 2

eng India and Belgium have completed 75 years of diplomatic relations.
hi भारत और बेि जयम ने राजनियक संबध
ं के  75 साल पूरे कर िलए ह।

Q­153 In which year Liberalized Exchange Rate Management System (LERMS) was instituted?
eng

01. March­1990 02. March­1991


03. March­1992 04. March­1993
05. March­1994
Q­153 उदारीकृत िविनमय दर बंध णाली (LERMS)  कस वष म लागू क गई थी?
hi

01. माच 1990 02. माच 1991


03. माच 1992 04. माच 1993
05. माच 1994
Solution for Question 153 Ans Key: 3

eng In 1992, Liberalized Exchange Rate Management System (LERMS) involving the dual exchange rate was instituted.
hi माच 1992 म उदारीकृत िविनमय दर बंध णाली (LERMS) म दोहरी िविनमय दर णाली लागू क गई थी।

Q­154 The headquarters of United Nations Development Programme (UNDP) is located in which city?
eng

01. Vienna 02. London


03. New York 04. New Delhi
05. Geneva
Q­154 संयु रा िवकास काय म (यूएनडीपी) का मु यालय कस शहर म ि थत है?
hi

01. िवयना 02. लंदन


03. यूयॉक 04. नई द ली
05. िजनेवा
Solution for Question 154 Ans Key: 3
eng The headquarters of United Nations Development Programme (UNDP) is located in New York, United States.
hi संयु रा िवकास काय म (यूएनडीपी) का मु यालय यूयॉक , संयु रा य म ि थत है।

Q­155 The United States has extended the New START Nuclear Treaty for five years with which nation?
eng

01. Japan 02. Russia


03. China 04. UK
05. USA
Q­155 संयु रा य अमे रका ने कस देश के साथ नई START परमाणु संिध को पांच साल के िलए बढ़ा दया है?
hi

01. जापान 02. स


03. चीन 04. यूके
05. यूएसए
Solution for Question 155 Ans Key: 2
eng The United States has extended the New START (Strategic Arms Reduction Treaty) nuclear arms control treaty with Russia for another five years. This
was announced by US Secretary of State Antony Blinken, who said that it made the world safer as unconstrained nuclear competition would endanger us
all.
hi संयु रा य अमे रका ने स के साथ नई START (साम रक श यूनीकरण संिध) परमाणु हिथयार िनयं ण संिध को और पांच वष के िलए बढ़ा दया है। इसक
घोषणा अमे रक िवदेश मं ी एंटनी लंकन ने क थी, िज ह ने कहा था क इसने दुिनया को सुरि त बना दया है य क अ ितबंिधत परमाणु ित पधा हम सभी को
खतरे म डाल देगी।

You might also like